Der Großteil des Universums ist unsichtbar. Die für uns normale Materie; die, aus der wir bestehen und all das, was wir um uns herum sehen können, macht nur knapp ein Viertel der gesamten Materie aus. Der Rest ist „dunkel“. Das bedeutet, dass wir zwar sehen und messen können, wie diese dunkle Materie die sichtbare Materie durch ihre Gravitationskraft beeinflusst. Wir können sie selbst aber nicht sehen. Heute gehen die meisten Wissenschaftler davon aus, dass die dunkle Materie aus einer noch unbekannten Art von Elementarteilchen besteht. Diese sogenannten WIMPs interagieren so gut wie kaum mit der normalen Materie (abgesehen von der Gravitation). Daher auch ihr Name: Weakly Interacting Massive Particles – WIMPs. Vielleicht entdecken wir sie demnächst. An großen Teilchenbeschleunigern wird danach gesucht und andere Experimente haben schon vielversprechende Hinweise geliefert. Aber wie das halt so ist mit Teilchen, die nicht mit normaler Materie interagieren, sind die verdammt schwer dingfest zu machen. Wenn die dunkle Materie wirklich aus WIMPs besteht, dann ist sie überall um uns herum, ohne das wir sie spüren. Die Teilchen würden sogar ständig unseren Körper durchdringen!

Was in diesem Fall passiert, haben Katherine Freese und Christopher Savage von den Universitäten Michigan und Stockholm in ihrer Arbeit „Dark Matter collisions with the Human Body“ untersucht.

Geht man von derzeit aktuellen Modellen aus, dann sind die Erde, die Sonne und der ganze Rest unserer Milchstraße in eine große Wolke aus dunkler Materie eingebettet. Genauso wie alle anderen Galaxien auch. Der gravitative Einfluss der dunklen Materie ist dabei nur auf sehr großen Skalen zu spüren; dann, wenn es zum Beispiel um die Bewegung von Galaxien in Galaxienhaufen geht. Unser irdischer Alltag wird von ihr nicht beeinflusst. Für WIMPs sind Erde und Menschen im Wesentlichen nicht existent. Gerade weil sie nicht mit der normalen Materie wechselwirken, können sie problemlos durch uns durchrauschen. Ein paar Milliarden WIMPs könnten in jeder Sekunde unseren Körper passieren, ohne das wir irgendwas davon merken.

Aber wir sind den WIMPs nicht völlig egal. Manchmal kann es doch vorkommen, dass ein WIMP mit einem normalen Atom wechselwirkt. Das kann natürlich irgendein Atom sein. Es gibt ja genug von ihnen. Ein Sauerstoffatom der Erdatmosphäre vielleicht oder ein Eisenatom tief im Erdkern. Oder aber auch ein Atom eines menschlichen Körpers. Es ist alles nur eine Frage der Statistik und genau die haben Freese und Savage durchgeführt. Wenn man von den bisher bekannten Vermutungen ausgeht und die bisher gewonnenen Beobachtungsdaten berücksichtigt: Wie viele der Milliarden und Abermilliarden WIMPs die in jeder Sekunde auf unseren Körper treffen, gehen nicht einfach durch sondern werden von den Atomen dort abgelenkt?

So sehen die Ergebnisse aus:

i-b4d37baf7fb26a926343d8e05a03e117-dark_matter_tab-thumb-500x462.jpg

In der ersten Spalte stehen die verschiedenen chemischen Elemente aus denen unser Körper besteht. Die zweite Spalte gibt an, welchen Anteil das Element an der Gesamtmasse des Körpers hat. Wenig überraschend sind es hauptsächlich Sauerstoff, Kohlenstoff und Wasserstoff. In der dritten und vierten Spalte findet man schließlich die Kollisionsrate pro Jahr. Es werden zwei Fälle unterschieden. „SI“ steht für „spin independent“ und „SD“ für „spin dependent“. Ganz simpel gesagt, findet im ersten Fall die Wechselwirkung des WIMPs über die Masse des Atoms statt, im zweiten Fall über dessen Spin. Verschiedene chemische Elemente unterscheiden sich durch ihren Aufbau; manche haben mehr Protonen und Neutronen, manche weniger. Protonen und Neutronen wechselwirken aber unterschiedlich stark mit den WIMPs, wenn man den SD-Fall betrachtet. Daher kommt es zu den unterschiedlichen Zahlen in Spalten 3 und 4.
Man sieht gleich, dass die Zahlen generell ziemlich niedrig sind. Ein Jahr hat grob 30 Millionen Sekunden. In jeder dieser Sekunde sausen ein paar Milliarden WIMPs durch unseren Körper. Und nur eine Handvoll davon wechselwirken mit den Atomen unseres Körpers. Es gibt 3 Kollisionen pro Jahr mit den Sauerstoffatomen und 22 mit denen des Wasserstoffs. Dann noch knapp 6 Kollisionen pro Jahr mit Phosphor. Insgesamt schaffen es gerade mal 35 WIMPs auf Atome in unserem Körper zu treffen.

Die Tabelle basiert auf den wahrscheinlichsten Annahmen über die Masse der WIMPs.
Da wir aber noch sehr wenig konkrete Daten über die Natur der dunklen Materie haben, sind natürlich viele Werte möglich. Das CRESST-Experiment beispielsweise lässt WIMPs mit anderen Eigenschaft zu und wenn man die Berechnungen mit ihnen wiederholt, dann könnten bis zu Hundertausend WIMPs im Jahr mit unserem Körper kollidieren. Das wären dann immerhin knapp 5 Kollisionen pro Stunde.

Egal welcher Wert korrekt ist (und vorausgesetzt, die WIMPs existieren überhaupt), wir können davon ausgehen, dass die Zusammenstöße mit unserem Körper nicht gefährlich sind. Immerhin gibt es die dunkle Materie schon, seit es das Universum gibt. Wenn solche Zusammenstöße für uns Menschen schädlich wären, dann würden wir gar nicht erst existieren.

Aus wissenschaftlicher Sicht ist die Arbeit von Freese und Savage nicht sonderlich dramatisch. Sie hat keine Konsequenzen, es lassen sich daraus keine beobachtbaren Effekte ableiten. Aber aus rein menschlicher Sicht ist es gut, dass jemand das mal konkret ausgerechnet hat. Wenn das Universum schon voll mit unsichtbarer Materie ist, die ständig durch uns hindurch rauscht, dann wollen wir zumindest wissen, wie egal wir dieser Materie genau sind. Jetzt wissen wir es. Ziemlich egal…

230 Gedanken zu „Wenn dunkle Materie mit dem menschlichen Körper kollidiert…“
  1. – Man kriegt eine Beule. Was sonst kann noch passieren, wenn ein Stein etwa auf den Kopf trifft? Die Steine leuchten nicht, wenn sie nicht beleuchtet werden. In der Nacht kann man über einen Stein stolpern und davon Verletzungen tragen. Und der meiste Raum im Weltall ist nicht beleuchtet.

  2. nur eine Handvoll davon

    Auch wenn ich glaube zu wissen, wie es gemeint ist (0-5), so ist in diesem Falle das gewollte kleine Teil mehr als das Ganze, da wohl nicht nur von mir zuerst das Volumen (mit den Händen schöpfen) verstanden werden dürfte.

    Davon (schon wegen der Unwichtigkeit) abgesehen: Ein angenehm unterhaltsamer post…

  3. Was passiert eigentlich bei solchen Kollisionen respektive Wechselwirkungen? Ändert sich dadurch der Spin? Kann ein Molekül zerfallen, weil die Position des Atoms durch die Wechselwirkung mit dunkler Materie verändert wird? Ändert sich da überhaupt etwas? Oder entspricht die Wechselwirkung eher der zwischen Satellit/Raumsonde (dunkle Materie) zu Planet (Atom)?

  4. @florian wie werden eigentlich die wimps in teilchenbeschleunigern gesucht? Hab mal drüber nachgedacht, die detektoren sind ja nicht für wimps ausgelegt. Sind da dann sowas wie ,,energie-lücken“ in den messdaten zu sehen?

  5. wir können davon ausgehen, dass die Zusammenstöße mit unserem Körper nicht gefährlich sind. Immerhin gibt es die dunkle Materie schon, seit es das Universum gibt. Wenn solche Zusammenstöße für uns Menschen schädlich wären, dann würden wir gar nicht erst existieren.

    Korrektur: wir können nur davon ausgehen, dass sie nicht zu schädlich sind. Eine (geringe) Erhöhung der Wahrscheinlichkeit, Krebs zu bekommen (um nur ein Beispiel zu nennen), führt noch nicht gleich zum Aussterben des Lebens an sich, dennoch wäre ohne diesen Effekt das Leben für einzelne Individuen länger. Man kann ja auch nicht sagen „Radioaktivität ist unschädich, denn es gibt natürliche Radioaktivität und trotzdem ist die Erde bewohnt“.
    Insofern wäre – wie ein Vorredner schrub – ganz interessant zu wissen, welche Art an Reaktionen erwartet werden (ohne das Paper jetzt komplett lesen zu müssen). Eine geringfügige Erwärmung ist dann doch etwas anderes als eine Kernumwandlung…

    Wieviele Neutrinos interagieren eigentlich pro Jahr mit dem menschlichen Körper? *Die* sind ja immerhin nachgewiesen 🙂

  6. In unserem Sternensystem sind die äußeren Planeten langsamer, als die inneren (was man ja auch recht gut mit klassischer Physik als Normalbürger nachvollziehen kann). Bei unserer Galaxie hingegen verhält es sich anders, hatte ich gelesen: Hier bewegen sich äußere und innere Sternensysteme gleich schnell um das Zentrum. Es wurde daraufhin spekuliert, dass hierfür die dunkle Materie verantwortlich sein könnte. Ist dies schon wieder vom Tisch? Das wäre ja doch ein „nicht ganz so große“ Skala, die die Entstehung von Galaxien beträfe.

  7. HalloThomas Vollmer, das ist überhaupt nicht vom Tisch. Und dieses Rotationsverhalten zeigt nicht nur unsere Galaxie, sondern wurde bei vielen 1000 andere Galaxien ebenfalls beobachtet. Ein schönes Bild das so eine Rotationskurve zeigt, findest du im Wikipedia-Artikel zu Dunkler Materie. Auch auf noch größere Skala, bei Galaxienhaufen wurde festgestellt, dass die Geschwindigkeiten, mit denen sich Galaxien „zu hoch“ sind. Es ist daher schon lange klar, und inzwischen auch zu, ich sag mal, 99,9% sicher, dass es Dunkle Materie gibt. Man weiß inzwischen auch woraus sie NICHT besteht: nicht aus Staub- und Gaswolken, nicht aus dunklen Sternen (sogen. MACHOS), nicht aus Neutrinos. Auch alternative Hypothesen, zum Beispiel mit einem anderen Gravitationsgesetz (Stichwort: MOND, gibt es auch einen Artikel bei Wikipedia) sind gescheitert. Als Kandidaten bleiben daher in erster Linie die oben erwähnten WIMPs übrig, und viele Experimente versuchen herauszufinden, welche Eigenschaften die WIMPs haben und was sie genau sind.

  8. DM agiert nur über Gravitation, also auch mit unseren Körpern nur gravitativ.
    Bei der angegebenen Rate sind einige wenige Atome des Körpers demnach ab und zu mal ein gaaaanz klein wenig stärker beschleunigt als ohne DM. Über die Richtung ist dabei ja noch gar nichts gesagt.
    Das dürfte wohl nichts bewirken, außer einer Temperatur-Erhöhung im nicht-meßbaren Bereich — das steht allerdings irgendwie im Gegensatz zum kalten Schauer, der mir manchmal über den Rücken läuft… vielleicht anti-WIMPS…;-)

  9. Interessant ist, wo man die durch DM erhöhte Gravitation findet, die zum unerwarteten Rotationsverhalten führt.
    Da gibt es offensichtlich 3 Radius-Bereiche um schwere Objekte herum:

    bis zu ~5pc: Außensterne von Galaxienkernen.
    bis zu ~1kpc: Außensterne von Kugelsternhaufen.
    bis zu ~1Mpc: Außensterne von großen Galaxien.

    Bei allen Bereichen werden um so größere Rotations-Geschwindigkeits-Abweichungen beobachtet, je weiter außen sich der jeweils beobachtete Stern befindet.
    Die angegebenen Grenzen werden nur von besonders großen Objekten tatsächlich erreicht, dann aber jeweils mit bis zu ~100-fach größerer Gravitationswirkung bei diesem Radius.

    Dazwischen scheint die Gravitationswirkung wieder kleiner zu werden.

    Wie kann das mit WIMPs erklärt werden?

  10. Hm, ich warte dann mal auf die ersten Esoteriker, die zum Beispiel das Völlegefühl nach zu viel schwerem Essen mit WIMPs erklären und das ultimative Gegenmittel (ein Umhang aus „energetisierter“ Bleifolie) dafür anbieten. 😀

  11. Nun ich werde dann wohl demnächst ins Geschäft mit DM abweisender Kleidung einsteigen. Da die Kollisionen der WIMPs „bekanntermassen“ der eigentliche Grund fürs Altern sind dürfte ich damit die neueste Lizenz zum Gelddrucken der Esoterikbranche gefunden haben 😀

  12. Ja, und inzwischen wird schon bestimmt eine Abschirmung von WIMPs patentiert…

    Die Leute, in diesem konkreten Fall – Freese und Savage, denken wahrscheinlich, dass sie wirklich was wissenschaftliches schaffen… Praktisch aber versorgen sie die Teleshops mit der Information, wie diese die Zuschauer auf besonders feine Art veralbern könnten.

  13. @engywuck· 11.04.12 · 04:17 Uhr

    Korrektur: wir können nur davon ausgehen, dass sie nicht zu schädlich sind. Eine (geringe) Erhöhung der Wahrscheinlichkeit, Krebs zu bekommen (um nur ein Beispiel zu nennen), führt noch nicht gleich zum Aussterben des Lebens an sich, dennoch wäre ohne diesen Effekt das Leben für einzelne Individuen länger. Man kann ja auch nicht sagen „Radioaktivität ist unschädich, denn es gibt natürliche Radioaktivität und trotzdem ist die Erde bewohnt“.

    Oder (um nur ein anderes Beispiel zu nennen), vielleicht gäbe es ohne diese Effekte gar kein menschliches Leben, da ein ein Faktor für Mutationen als Beschleuniger der Evolution gefehlt hätte. Wer weiß?

    […] Wieviele Neutrinos interagieren eigentlich pro Jahr mit dem menschlichen Körper? *Die* sind ja immerhin nachgewiesen 🙂

    Keine Ahnung, aber ich habe mal spaßeshalber überschlagen, wieviele C14-Atome im menschlichen (und jedem anderen lebenden) Körper pro Sekunde zerfallen.
    Vielleicht erlaubt Dir das eine Einschätzung, ob die paar 10 gravitativen Wechselwirkungen pro Jahr wohl doch vernachlässigbar sein könnten? (manchmal kann ein wenig Rechnen doch ganz ‚erhellend‘ sein)

  14. Vielleicht erscheine ich altertürmlich, aber mit den Steinen ist es ziemlich ernst. Man geht von der Existenz einer Wolke aus… festen Objekten wie Steinen außerhalb des Sonnensystems…. https://de.wikipedia.org/wiki/Oortsche_Wolke Man kann sie zwar nicht sehen, aber die Bahnen mancher Kometen deuten daraufhin, dass sie existiertt…

    Wenn ein Raufschiff wie Enterpreis wagt das Sonnensystem zu verlassen, werden seine Sensoren anzeigen: „Autsch… autscht“ usw., denn denn die Steine wie ein Regen auf es schlagen…

    Was die Rotationskurven betrifft, oszillieren sie:

    https://worlov.wordpress.com/2010/12/15/oszillierende-rotationskurven-und-dunkle-materie/

    Haben Sie andere Erklärung wie ich?

  15. Haben Sie andere Erklärung wie ich?

    N’Abend, Walter Orlov, da fehlt etwas am Ende, mögliche Ergänzungen wären zum Beispiel

    • am günstigsten nach Unna komme
    • 3 bis 4 Kilo abnehmen könnte
    • mir Grundlagen über Astronomie erarbeiten könnte
    • einschlafe

    Bitte erst entscheiden, dann auf Antwort hoffen.

    Falls um etwas wie den dritten Punkt geht, können wir jetzt die Erläuterungen über die Dichte der Oortschen Wolke erst einmal aufschieben.

  16. Wusste ich’s doch:
    Das Universum ist mit Weicheiern durchsetzt!
    Warum nur muss ich ständig an Manowar denken?
    („Wimps and posers: Leave the hall!“)
    😀

  17. Oortsche Wolke darf durchaus sehr kleine Dichte haben. Am Ende geht es doch um Dichte x Raum und Raum gibt es im Weltall jede Menge. Außerdem befindet sich unser Sonnensystem gerade zwischen den Spiralarmen, also im Gebit mit geringster Massendichte. Und bei meinen Berechnungen ging ich davon aus, dass unsere Umgebung einigermaßen sauber ist und dementsprechend die Massendichte allein durch die Sternendichte bestimmt wird.

    Aber anscheinend möchte noch keiner wirklich über die Oszillationen in den Rotationskurven der Spiralgalaxien nachdenken – die Entdeckung der neuartigen Teilchen würde bestimmt mehr Ansehen bringen, als Geplauder über gemeine Steine im Weltall 🙂

  18. @Orlov: „Wenn ein Raufschiff wie Enterpreis wagt das Sonnensystem zu verlassen, werden seine Sensoren anzeigen: „Autsch… autscht“ usw., denn denn die Steine wie ein Regen auf es schlagen…“

    Sie sind auf die falschen Darstellungen in den Sci-Fi-Filmen reingefallen. Um gegen einen Asteroiden zu prallen, muss man sich schon sehr anstrengen: https://www.scienceblogs.de/astrodicticum-simplex/2010/07/die-grosse-der-asteroiden.php

  19. Schon die Oberfläche der Asteroiden verrät, dass sie von kleineren Objekten als sie selbst bombardiert wurden. Heutige Teleskopen sind wahrscheinlich nicht empfindlich genug, um sie ausfindig zu machen. Hoffentlich bringt der Nachfolger von Hubble mehr Klarheit in die Sache.

  20. @Orlov:

    Außerdem befindet sich unser Sonnensystem gerade zwischen den Spiralarmen, also im Gebit mit geringster Massendichte.

    Woher das „also“? Wie hoch ist denn der Unterschied zwischen „hoher“ und „geringer“ Massendichte in den jeweiligen angesprochenen Gebieten?

  21. Ich habe schnell auf meiner Seite nachgeschaut – ca. eine Großordnung. Weil man aber behauptet, dass die Sternendichte von der Spiralstruktur kaum beeinflusst wird, müsse dann in den Spiralarmen noch 10-fach größere Masse als die Masse der Sterne dazu kommen. Mit den unsichtbaren Teilchen würde es wohl nicht klappen – sie werden uns die Sicht etwa zum Zentrum der Milchstraße wohl nicht sperren können… dafür aber der sogenannte „Staub“. Aber ob es wirklich nur der Staub ist, oder handelt es sich um die Sammlungen von größeren Objekten wie Steine, Asteroiden, Planetoiden und Planeten, kann man durch Teleskop nicht sehen. Deshalb vermute ich, dass es doch die Gesteinsbrocken verschiedener Größe sind, die uns die Sicht vernebeln.

  22. @zetaOri: Danke für die Berechnungen – ab jetzt esse ich nur noch gesunde (da nicht radioaktive) aus Erdöl hergestellte Nahrungsmittel, da ist nur C13 (und C12) drin. :-p

    Wobei… das wäre mal ne Marktlücke für gewisse Esos – radioaktiv abgereicherte Nahrungsmittel mit isotopengetrenntem Kalium und Pflanzen, die auf künstlichem Substrat in Treibhäusern mit kontrolliertem CO2 wachsen. Natürlich wird nur mit Tritiumabgereicherten und von Uran etc befreitem Wasser gewässert (das herausgefiterte Tritium nehmen uns Iran und so sicher gerne ab). Damit dürften die meisten Strahlenquellen in der Nahrung beseitigt sein… Dann das ganze in Verpackungen ohne EAN-Barcode stecken, damit die nicht nachträglich durch den Laser im Supermarkt verstrahlt werden…
    Oder gibt’s das etwa schon? (vorstellen könnt‘ ich’s mir)

  23. Florian Freistetter, irgendwo haben Sie gesagt, Realität ist das was übrig bleibt wenn man aufhört daran zu glauben (oder so ähnlich).

    Was, wenn man aufhört daran zu glauben dass die Gravitationstheorie im gesamten Universum gültig ist, ohne Skalierungsprobleme (damit meine ich die Art von Problem, an dem z.B. die Newton’sche Version gescheitert ist)?

    Dann bleibt keine Dunkle Materie übrig, weil man die perfekte Gültigkeit der Gravitationstheorie braucht um sie zu berechnen.

    Folglich ist die Dunkle Materie irreal. Eine aussersinnliche Wahrnehmung (was ein sehr schöner Name für berechnete Artefakte ist).

    Zweifelhaft, ob man sie jemals finden kann. Lustig aber, dass die Wissenschaftsvermittler trotzdem schon mit vollster Überzeugung so tun als ob das alles eine Darstellung der Realität wäre. Scheint, als wollten die Wissenschaftsvermittler genauso grandios am Realitätsbezug scheitern wie die Glaubensvermittler.

  24. @Realistischer: „Dann bleibt keine Dunkle Materie übrig, weil man die perfekte Gültigkeit der Gravitationstheorie braucht um sie zu berechnen. Folglich ist die Dunkle Materie irreal. „

    Ähm – ne, so funktioniert das nicht. Ich hab das hier erklärt: https://www.scienceblogs.de/astrodicticum-simplex/2008/12/neptun-vulkan-und-die-dunkle-materie.php
    Man hat ja schon probiert, die dunkle Materie mit einem anderen Gravitationsgesetz zu erklären. Aber das funktioniert nicht.

  25. @Orlov: „Deshalb vermute ich, dass es doch die Gesteinsbrocken verschiedener Größe sind, die uns die Sicht vernebeln. „

    Ich vergaß: Sie interessieren sich ja nicht wirklich für Fakten… Dann lassen wir die Diskussion am besten.

  26. @Realistischer, der Vollständigkeit halber:

    Florian Freistetter, irgendwo haben Sie gesagt, Realität ist das was übrig bleibt wenn man aufhört daran zu glauben (oder so ähnlich).

    Hat Florian bestimmt auch mal zitiert, im Original stammt’s von Philip K. Dick: „Reality is that which, when you stop believing in it, doesn’t go away.“

  27. Naja, nur weil die MOND-Theorie nicht passt, ist das noch lange keine Bestätigung für die DM. Es ist noch nicht einmal ein Argument für die DM, nur eins gegen MOND.

    Zurück zu meinem Argument: die DM wird BERECHNET, und zwar mittels dem Glauben dass die Gravitatiosntheorie UNBESCHRÄNKTE GÜLTIGKEIT hat. Dieses Argument wurde nicht angetastet. Es bleibt daher, gemäß Definition von Realität, dass die DM irreal ist.

    Entweder man steht zur Definition und zieht die Konsequenzen, oder man gibt die Definition auf. Grundlegende Regel der Logik.

  28. Zu Deinen drei Abschnitten, Realistischer: Der erste widerlegt eine Aussage, die hier nicht gemacht wurde. Strohmann. Der zweite ist gequirlter Blödsinn. Der dritte klingt bedeu­tungs­schwanger, ist aber irrelevant, weil es hier nicht um Definitionen geht.

    Ich weiß ja nicht, wie es in Deiner Realität so zugeht, doch in der von mir erlebten wer­den in den NaWis Beobachtungen gemacht, eine Erklärung gesucht und deren Gültigkeit ge­tes­tet. Solange bis etwas widerlegt oder durch etwas Besseres abgelöst wird, bleibt es als Erklärungsmodell, als Theorie erhalten.
    Insofern wurde nicht geglaubt, daß die GTheorie richtig ist, sondern wohlbegründet an­ge­nommen. Unbenommen der Möglichkeit, daß dies anders sei — doch jegliche Versuche in der Richtung sind bisher mehr oder weniger grandios gescheitert.
    Nenne einen Widerspruch des Normal-DM-DE-Modelles zu gemachten Beobachtungen, biete optional ein neues Modell und werde ein Nobelstar. Bis dahin ist es keineswegs eine sinn­volle Alternative, herumzusülzen und pseudowissenschaftliche Phrasen zu dreschen.

  29. @rolak Das Beste an der DM-„Theorie“ ist dass man sie nicht widerlegen kann, weil die DM nicht beobachtbar ist. Eigentlich ist allein das schon ein Ausschliessungsgrund, unwiderlegbare „Theorien“ sind unwissenschaftlich.

  30. @realistischer: „Es bleibt daher, gemäß Definition von Realität, dass die DM irreal ist“

    Ich vermute zwar stark das die Disskussion mit dir nichts bringt, aber trotzdem:

    Wir haben folgende Situation. Wir BEOBACHTEN, dass sich die Sterne und Galaxien nicht so bewegen, wie sie es sollen, sondern so, als ob da noch mehr Materie ist, die man nicht sehen kann. Wir BEOBACHTEN den Gravitationslinseneffekt dieser Materie. Usw. Es existieren jede Menge Beobachtungen, die irgendwie interpretiert werden müssen. Es wurde schon probiert, dieses Problem durch verschiedene Abänderungen des Gravitationsgesetzes (das anderweitig aber wunderbar funktioniert) zu lösen. Das klappt nicht. Man hat stattdessen Unmengen an unabhängig voneinander gewonnenen Beobachtungsdaten, die uns den Effekt von dunkler Materie zeigen. Wie sollten die Wissenschaftler deiner Meinung nach also vorgehen?

  31. Wie gesagt, Realistischer, es ist keine Alternative, pseudowissenschaftliche Phrasen zu dreschen. Ganz im Gegensatz zu Deinem Geschwalle kann DM beobachtet werden – und zwar wie alles andere auch: Durch Wechselwirkung mit Anderem (speziell, aber nicht unbedingt dem Meßinstrumentarium). Denn wie Du anscheinend in der maximal 3/4-Stunde zwischen Lesen und Schreiben schon wieder vergessen hast, ist der Ausgangspunkt eine Beobachtung.

    Den einzig sinnvollen Grund gegen DM hast Du übrigens auch vergessen: Widerspruch oder besseres Modell. Bis dahin, wie wäre es mit etwas deutlich Produktiverem, Däumchen­drehen zB?
    Übrigens zeigt eine realistische Betrachtung, daß Dein nick sehr schlecht gewählt ist.

  32. @Realistischer: „Das Beste an der DM-„Theorie“ ist dass man sie nicht widerlegen kann, weil die DM nicht beobachtbar ist.“

    Wie gesagt. Das ist falsch. Es gibt jede Menge Beobachtungen. Diese Beobachtungen müssen irgendwie interpretiert werden. Oder schlägst du vor sie einfach zu ignorieren.

  33. @Florian Freistetter: Die Beobachtungen interpretieren muss bzw. besser will man natürlich, aber trotzdem ist DM nichts das man zwingend akzeptieren müsste. Ich erlaube mir daher bis auf weiteres, sie nicht zu akzeptieren – das ermöglicht mir eine kritische Sicht.

    Bei der Gelegenheit eine Frage: wird in den Computersimulationen welche die Gravitation berechnen auch die Ausbreitung der Gravitation (Gravonen) berücksichtigt? Denn je größer die Distanzen desto relevanter wird das bzw. desto größer würde der Rechenfehler wenn man es weglässt.

  34. @Realistischer: „wird in den Computersimulationen welche die Gravitation berechnen auch die Ausbreitung der Gravitation (Gravonen) berücksichtigt?“

    Natürlich. Und es heisst „Gravitonen“.

    „Ich erlaube mir daher bis auf weiteres, sie nicht zu akzeptieren – das ermöglicht mir eine kritische Sicht.“

    Kannst du ruhig machen. Aber wenn du außer „Ich finde DM doof und ignoriere alle Beobachtungsdaten“ nichts zu bieten hast; keine Alternativerklärung zum Beispiel, dann sei nicht böse, wenn man dich nicht ernst nimmt.

  35. @Florian Freistetter: Meine Beobachtungsdaten includieren die Wissenschaftsgeschichte, die zeigt dass schon viele Theorien korrigiert werden mussten, und dass schon viele Vorwände produziert wurden um Theorien doch noch etwas länger halten zu können (wobei sich natürlich immer erst hinterher herausstellt was nachhaltig ist und was nur erfolgloser Rettungsversuch war).

    Solange man DM nicht wirklich messen kann sondern nur über abenteuerliche Umwege berechnen, finde ich es garnicht doof dem kritisch gegenüber zu stehen. Umso mehr als die populärwissenschaftlichen Darstellungen allermeist nur unkritische Leistungsschauen sind.

  36. @Realistischer: „Solange man DM nicht wirklich messen kann sondern nur über abenteuerliche Umwege berechnen,“

    Man kann die Auswirkungen der DM messen. Und daran ist nichts abenteuerlich.

  37. Man kann auch die Auswirkungen Gottes in der Welt erkennen. Ein Blick in die Geschichte der Menschheit zeigt, im Großen und Ganzen wendet sich alles, trotz vieler Probleme und Krisen, immer wieder zum Guten. Das ist Gottes Einfluss, der ist ganz klar sichtbar.

    Wenn man daran glaubt.

    Es könnte natürlich auch nur eine Eigenschaft der Lebewesen bzw. der Evolution sein, welche dazu führt dass die destruktiven eher aussterben, und eine zufälligerweise günstige Konstellation der Erde, welche diese lange anhaltende Entwicklung ermöglicht hat.

    Genauso muss das beobachtete Verhalten der Materie nicht notwendiger Weise auf eine nicht an sich erkennbare DM zurückgeführt werden sondern es könnten bislang noch unbekannte Eigenschaften der erkennbaren Materie selbst die Ursache sein.

    Die man aber nur findet wenn man sie auch sucht.

    Jedenfalls glaube ich letzteres nun einmal eher als den DM-Ansatz. Die DM ist mir zu ungreifbar, zu Gott-ähnlich in dem Sinn dass man nur eine Wirkung aber nicht das Verursachende beobachten kann. Eigentlich ist das ein typisches Esoterik-Muster.

    Mit besten Grüßen von der Unsichtbaren Hand,
    Ihre Realität.

  38. @Realistisch: „Man kann auch die Auswirkungen Gottes in der Welt erkennen“

    Ok, jetzt wird es vollends offtopic und absurd. Also lassen wir die Diskussion bleiben. Ich hab ja kein Problem, die Sache mit der DM zu erklären. Aber wenn jemand nicht verstehen will, dann macht das keinen Sinn…

  39. @Realistischer
    Hmm du merkst gar nicht, daß du dir selbst widersprichst?
    1.

    Man kann auch die Auswirkungen Gottes in der Welt erkennen. Wenn man daran glaubt.

    und
    2.

    edenfalls glaube ich letzteres nun einmal eher als den DM-Ansatz. Die DM ist mir zu ungreifbar, zu Gott-ähnlich in dem Sinn dass man nur eine Wirkung aber nicht das Verursachende beobachten kann.

    Hääääh? Also du glaubst an Gott, weil man seine Wirkungen beobachten kann, aber nicht an die DM, weil man nur ihre Wirkungen beobachten kann und das sei dir zu Gott-ähnlich.
    Hallo? Wie wäre es mal mit dem Nachdenken über diese allein von dir getroffenen Aussagen und ihre innere Widersprüchlichkeit?
    Darüber hinaus sollst du nicht einfach glauben, es geht hier um Wissenschaft, nicht um Glaube.

  40. Ich glaube nicht an Gott. Und an die DM glaube ich auch nicht. Das war bitte ein Vergleich der Ansätze. Wenn das nicht rüberkommt, dann verstehen wir uns einfach nicht. Sorry für die Überforderung.

  41. @Realistisch: „Und an die DM glaube ich auch nicht. „

    Du scheinst zu denken, dass „glauben“ in der Wissenschaft eine Rolle spielt. Das ist wohl dein Problem…

  42. Mir ist schon klar, dass da einfach was bewobachtet wird, was nicht so richtig passt und dass der DM-Ansatz der bis jetzt am erfolgversprechendste ist. War ja mit der Relativitätstheorie nicht anders, aber da konnte/kann man ja sagen, dass die Theorie nicht passen würde, wenn man bspw. die Zeitdilatation bei bewegten Uhren nicht sehen/messen könnte.
    Gibt es theoretische Szenraien/Vorhersagen aus der DM-Theorie, die diese falsifizieren könnte?

  43. @Realistischer
    Dann war dieser Absatz also als Satire von dir gedacht?

    Man kann auch die Auswirkungen Gottes in der Welt erkennen. Ein Blick in die Geschichte der Menschheit zeigt, im Großen und Ganzen wendet sich alles, trotz vieler Probleme und Krisen, immer wieder zum Guten. Das ist Gottes Einfluss, der ist ganz klar sichtbar.

  44. @Adent: Eigentlich ist das eine Beschreibung dessen was sich die Gläubigen denken (so wie ich es mir erkläre). Wenn man grundsätzlich pessimistisch ist, und immer wieder überrascht oder verwundert wenn doch gutes geschieht, sodass man meint dass das nicht der natürliche Lauf der Welt war sondern dass da ein guter Einfluss von „oben“ hinzu kam – dann kann man das Gott nennen. Und dessen Werk ist dann auch klar erkennbar, als Differenz zur pessimistischeren Erwartung. Gott stellt, so gesehen, einen Erwartungsfehler bzw. dessen „Lösung“ dar.

    Weil die DM auch eine Differenz zu einer nicht erfüllten Erwartung darstellt, bin ich auf diesen Vergleich gekommen, und deshalb bin ich auch pessimistisch was die DM betrifft. Aber vllt. geschieht ja noch ein Wunder und Gott erschafft die DM 😉

  45. @Florian Freistetter: nochmal zurück zur Frage nach der Simulation der Gravitation – die bzw. deren Ergebnisse sind ja die Grundlage für alle weiteren Überlegungen zur DM. Hätten Sie zufällig einen Link oder evtl. einen Blog-Post der zeigt was da eigentlich berechnet wird (Gravitationskräfte, Raumkrümmung, Gravitonendichte) und was nicht (welche Auflösung hat das, wie stellt man Raumkrümmung dar falls überhaupt)? Gravitonen zu berücksichtigen vereinfacht ja die Berechnung nicht gerade, da hat man sich sicherlich einige Vereinfachungen einfallen lassen müssen.

  46. @SCHWAR_A: ich dachte an eine Simulation die Gravitonen mit berücksichtigt. Man braucht entweder die 4. Dimension um die Ausbreitungsgeschwindigkeit annähernd berechnen zu können, oder gleich ein Modell der Gravitonenbewegung selbst in der Simulation. Und auf jeden Fall mehr Rechenpower.

  47. @Orlov: „Auch wenn wir die Planeten im galaktischen Staub nicht direkt beobachten können, können sie im Staub versteckt sein 😉 „

    Sorry, aber mit ihrer enormen astronomischen Unkenntnis machen sie sich höchsten lächerlich. Lesen sie mal ein paar Grundlagenbücher über Astronomie, dann fällt ihnen vielleicht auf, warum sie hier niemand ernst nimmt.

  48. @Realistischer:
    Gravitonen sind aber doch nur die (gedachten) Austauschteilchen. Die Wirkung der Gravitation selbst ist bereits durch die Gleichung des Feldes, mit oder ohne Modifikationen, viel einfacher berechenbar, als wenn Du für jedes einzelne Austauschteilchen (wie viele müßten das wohl sein??) eine Berechnung machen müßtest. Das Ergebnis muß dasselbe sein, sonst wäre ja was faul an der Austauschteilchen-Idee…

    Wieso nimmst Du eigentlich an, daß sich Gravitonen mit einer anderen Geschwindigkeit als c bewegen?

  49. @Florian Freistetter
    Sie wollen doch nicht behaupten, dass der Stern selbst Staub und Planeten ausgespuckt hat… Sie konnten nur vom Außen, aus dem Weltall, zum Stern kommen.

    Und mit der Wichtigtuerei können Sie ruhig aufhören – das wirkt auf mich nicht.

  50. @Orlov: „Sie wollen doch nicht behaupten, dass der Stern selbst Staub und Planeten ausgespuckt hat… „

    Nein. Will ich nicht und habe ich nicht.

    „Sie konnten nur vom Außen, aus dem Weltall, zum Stern kommen.“

    Wie gesagt: Lesen sie ein paar Bücher über Astronomie. Dann verstehen sie auch, wo der interplanetare Staub herkommt.

  51. Bestimmt meinen Sie die Kollisionen von Asterioden. Gib es eine einzige Videoaufnahme, die diese Hypothese auch belegen könnte? Aus meiner Sicht könnte interplanetarer Staub, wenigstens ein Teil davon, durchaus eine Art Niederschlag aus dem Weltall sein. Die Anziehungskraft der Sonne hört bei höheren Distanzen doch nicht auf.

  52. @Orlov: „Gib es eine einzige Videoaufnahme, die diese Hypothese auch belegen könnte?“

    Es gibt jede Menge Beobachtungsdaten die das belegen. Aber da sie ja die Realität regelmäßig verleugnen (und sich weigern die Grundlagen der Astronomie und Physik zu verstehen), werden die sie auch nicht überzeugen. Und jetzt habe ich genug Zeit auf sie verschwendet. Viel Spaß noch in ihrer eigenen Welt.

  53. @SCHWAR_A: Kommt ganz auf die Feldgleichungen an. Die gravitativen Kräfte auf ein Masseobjekt zu einem Zeitpuntk t werden NICHT von den anderen Masseobjekten zu diesem Zeitpunkt t bestimmt, sondern von irgendwann vorher, t-xi, wobei xi von der Entfernung zum Objekt i abhängig ist bzw. der Zeit die die Gravitonen zur Überbrückung dieser Entfernung brauchten.

    Man braucht dafür also ein 4D-Feld, falls man nicht die Gravitonen selbst darstellt. Und sowas wie einen Dopplereffekt wird’s ja auch geben, ausserdem bewegen sich auch die Gravitonen im gekrümmten Raum…

    Es stellt sich die Frage wie genau man das überhaupt simulieren kann, bzw. ob man den Rechenfehler einschätzen kann, den man für eine praktikable Laufzeit in Kauf nehmen muss. Der wiederum wirkt als Hebel auf die Ungenauigkeiten der DM, falls es die gibt.

  54. @Realistischer:
    „Die gravitativen Kräfte auf ein Masseobjekt zu einem Zeitpuntk t werden NICHT von den anderen Masseobjekten zu diesem Zeitpunkt t bestimmt, sondern von irgendwann vorher, t-xi, wobei xi von der Entfernung zum Objekt i abhängig ist…“

    Aber genau das tun doch bereits die Gleichungen unter Berücksichtigung der SRT und der ART.

    Mir scheint, Du möchtest das Rad (hier die ART) neu erfinden, in dem Du das zeitliche Verhalten (Laufzeit gemäß Entfernung mit Lichtgeschwindigkeit) in statische Felder (hier Newton’sche Gravitation) einführst. Wenn Du diese Formel (eine Lösung der Vakuum Feldgleichung) analysierst, stellst Du fest, daß all Deine „4-Dimensionalität“ bereits eingebaut ist… Du würdest also durch Deine „Simulation aller Gravitonen“ genau dasselbe Ergebnis erhalten…

    Schau‘ Dir dazu mal bitte die sehr gute „Serie Raumzeitkrümmung und Relativitätstheorie“ von Martin Bäker, Hier wohnen Drachen, an…

  55. @Alderamin:
    „…for distances of 100 million light-years.“

    DE macht sich doch erst ab ~7 Mrd Lichtjahren (z~0.5) bemerkbar, also erst etwa 70 mal weiter. Wie ist der Zusammenhang zu MOG gemeint?

  56. @SCHWAR_A

    Es geht darum, dass die Beobachtungen eine Verifzierung der ART auf einer Skala von 100 Millionen LJ erlauben. Das mag für Dark Energy noch nicht relevant sein, ist es aber für MOG vs. DM.

    Bisher war die ART laut Artikel nur für Entfernungen innerhalb des Sonnensystems einwandfrei durch Messungen belegt.

  57. @SCHWAR_A: Die vorhin von Ihnen gepostete Simulation kann das aber nicht, die hat keine 4. Dimension und kann daher keine Ausbreitungsgeschwindigkeit berücksichtigen. Da wird also kein Quanten- sondern nur ein klassisches Kraftfeld berechnet. Vllt. wussten Sie das eh, aber wenn es als Antwort auf die Frage nach einer Quantenfeldsimulation kommt, dann nehme ich das Gegenteil an.

    Darüber hinaus bleibt noch, dass man, wenn man via 4D-Feldgleichungen die Gravitonen wegabstrahiert, Effekte wie jenen dass die ihrerseits von Raumkrümmung beeinflusst werden nicht darstellen kann. Das könnte bei einer Galaxienscheibe schon eine kleine(?) Extra-Gravitation hin zum Zentrum erzeugen.

  58. @Realistischer
    „Effekte wie jenen dass die ihrerseits von Raumkrümmung beeinflusst werden nicht darstellen kann. “
    Das ist falsch, der Effekt ist selbstverständlich in der Feldgleichung der ART enthalten. Gravitonen sind ja letztlich nur die Linearisierung der quantisierten Form dieser Gleichung. (Annahme einer kleinen Abweichung der metrik von 1 und Quantisierung dieser Abweichung mit den üblichen Regeln.)
    Genauso wie für makroskopische Systeme aus der normalen Quantenmechanik die gwohnten Gleichungen der Mechanik herauskommen, ist der klassische Grenzfall der (noch zu erstellenden) Quantengravitation notwendigerweise die ART, sonst wäre die Theorie falsch.

    „Glauben“ Sie eigentlich, dass es neutrinos gibt? Die kann man auch nicht „sehen“, genauso wie die dunkle Materie.

  59. @MartinB: Der Übergang zwischen Wissen und Glauben ist schleifend, je weniger und unsicherer die Fakten desto mehr Glaubensanteil, oder Spekulationsgrad, oder wie man das Nicht-Wissen so benennen möchte.
    Wissen Sie schon dass man eine Quantengravitation erstellen kann welche die ART perfekt realisiert (und, wichtiger, die experimentell bestätigbar ist), oder glauben Sie es noch?

  60. @Realistischer
    „je weniger und unsicherer die Fakten desto mehr Glaubensanteil, oder Spekulationsgrad, oder wie man das Nicht-Wissen so benennen möchte.“
    Deswegen habe ich ja nach den Neutrinos gefragt – deren Nachweis ist auch immer ziemlich indirekt. Weil mir nicht so klar ist, wo Sie die Grenze ziehen und sagen „DM ist jenseits davon“.

    „Wissen Sie schon dass man eine Quantengravitation erstellen kann welche die ART perfekt realisiert“
    Nein, das weiß ich nicht. Aber *Sie* haben ja angefangen, nach Gravitonen zu fragen und so die Annahme einer Quantengravitation ins Spiel gebracht, ich habe nur argumentiert, dass das eine unnötige Annahme ist.
    *Wenn* es eine Quantengravitation gibt, dann muss sie die ART im Grenzfall enthalten (weil die ART ja durch Messungen sehr genau bestätigt ist). Wenn es keine QG gibt – tja, dann wird’s knifflig für die Physik.

  61. SCHWAR_A·
    16.04.12 · 16:53 Uhr

    @Alderamin:
    Gab es denn da einen Widerspruch zwischen MOG in Form von TeVeS und ART?

    Das entsprechende Paper („…an interpretation of these results in terms of dark energy
    and modified gravity models is presented in Samushia et al.
    (2012)…“, eine Zeile aus https://arxiv.org/abs/1203.6594, auf welches man stößt, wenn man den Links im S&T-Artikel folgt) ist leider noch nicht online.

    Aaaaber: Wenn laut MOG oder MOND die Rotationskurven von Galaxien nicht durch Newton bzw. die ART (außer mit DM, die jedoch von MOG/MOND gerade bestritten wird) beschrieben werden, dann muss ja ein anderes Abstandsgesetz als 1/r² gelten, und wenn sich das auf Galaxiendurchmessern von ein paar tausend LJ auswirkt, dann würde man es doch erst Recht auf Abständen von 100 MLJ bemerken müssen, und das lässt sich durch Statistiken über die Radialgeschwindigkeiten verifizieren (die Transversalgeschwindigkeiten und damit die tatsächliche Bewegung durch den Raum kann man leider nicht beobachten). Mal gespannt, wann das Paper kommt.

  62. @MartinB: Nein, die Quantengravitation ist nicht unnötig. (Glauben Sie nicht, ich schon.)

    Eine ART kann selbstverständlich NICHT Effekte beinhalten die sich erst aus der Berücksichtigung der Quantenebene ergeben, ansonsten müsste sie selbst eine Quantentheorie sein. Und aus genau diesem Grund wird die Quantengravitation keine perfekte Realisierung der ART sein – die ART aber auch nicht ganz falsch, sondern eine Annäherung.

    Übrigens: „Genauso wie für makroskopische Systeme aus der normalen Quantenmechanik die gwohnten Gleichungen der Mechanik herauskommen“ — auch das gilt nur Näherungsweise.

  63. @Realistischer

    Die gravitativen Kräfte auf ein Masseobjekt zu einem Zeitpuntk t werden NICHT von den anderen Masseobjekten zu diesem Zeitpunkt t bestimmt, sondern von irgendwann vorher, t-xi, wobei xi von der Entfernung zum Objekt i abhängig ist bzw. der Zeit die die Gravitonen zur Überbrückung dieser Entfernung brauchten.
    Man braucht dafür also ein 4D-Feld

    Aus dem Genannten folgt eigentlich nur, dass man ein retardiertes Potential braucht.

  64. @Realistischer

    Die gravitativen Kräfte auf ein Masseobjekt zu einem Zeitpuntk t werden NICHT von den anderen Masseobjekten zu diesem Zeitpunkt t bestimmt, sondern von irgendwann vorher, t-xi, wobei xi von der Entfernung zum Objekt i abhängig ist bzw. der Zeit die die Gravitonen zur Überbrückung dieser Entfernung brauchten.
    Man braucht dafür also ein 4D-Feld

    Aus dem Genannten folgt eigentlich nur, dass man ein retardiertes Potential braucht.
    Ein „4D-Feld“ oder Gravitonen kann man allein daraus jedenfalls nicht schussfolgern.

    Und aus genau diesem Grund wird die Quantengravitation keine perfekte Realisierung der ART sein

    Niemand weiß, was du mit „perfekter Realisierung“ überhaupt meinst.

    Dass eine Quantengravitation alle experimentell bestätigten Vorhersagen der ART als Grenzfälle enthält, aber ausgerechnet die dunkle Materie auf magische Weise aus den Gleichungen verschwinden lässt, ist jedenfalls extrem unwahrscheinlich.

    Übrigens: „Genauso wie für makroskopische Systeme aus der normalen Quantenmechanik die gwohnten Gleichungen der Mechanik herauskommen“ — auch das gilt nur Näherungsweise.

    Wenn du MartinBs Satz zu Ende gelesen hättest, hättest du dir deinen ganzen letzen Beitrag sparen können.
    Der Satz lautete nämlich:

    Genauso wie für makroskopische Systeme aus der normalen Quantenmechanik die gwohnten Gleichungen der Mechanik herauskommen ist der klassische Grenzfall der […] Quantengravitation notwendigerweise die ART, sonst wäre die Theorie falsch.

    Das wurde im MartinBs letztem Beitrag doch sogar noch einmal wiederholt?

    Ist jetzt nicht böse gemeint, aber bevor du weiter ausführst, warum die ART und die Annahme von dunklen Materie Unsinn sind, solltest du dich erstmal damit befassen, was diese Konzepte überhaupt bedeuten, was sie leisten und wie man darauf gekommen ist.
    Dann merkst du vielleicht, dass man einen deutlich geringeren Glaubensanteil braucht, als du annimmst.
    Das ist allgemein ein guter Tipp: Bevor man anderen Nicht-Wissen vorwirft, sollte man selbst das Thema allermindestens ansatzweise verstanden haben.

  65. @Realistischer
    „Eine ART kann selbstverständlich NICHT Effekte beinhalten die sich erst aus der Berücksichtigung der Quantenebene ergeben“
    Angewandt auf unser Problem hier heißt das, dass Sie die Idee haben, dass eine Quantengravitation die ART genau so modifiziert, dass die Notwendigkeit der dunklen Materie wegfällt, aber alle sonstigen Beobachtungsergebnisse unverändert vorhergesagt werden? Die Quantengravitation soll also makroskopische Auswirkungen auf Galaxienskala haben, aber keine auf der Längenskala von Planetensystemen oder Doppelpulsaren?

    Das halte ich für extremst unwahrscheinlich.

    PS: Auf die Neutrino-Frage hätte ich wirklich gern eine Antwort.

  66. @MartinB: Ob die Notwendigkeit einer DM wegfällt wird sich zeigen (es gibt ja eine Menge an Planeten, Staub und usw. die einfach nicht leuchten und daher schwer einzuschätzen sind – aber ob man das dann noch DM nennt?).

    Dass die Quantengravitation erst ab großen Distanzen auffällig wird kann man sich leicht denken, wenn man sich vorstellt „wie die Quanten fliegen“, auch durch gekrümmten Raum.

    Zu den Neutrinos bin ich sozusagen neutral, das motiviert mich nicht um dafür oder dagegen zu argumentieren. Was man findet, das hat man gefunden, der Rest bleibt Spekulation.

  67. „Dass die Quantengravitation erst ab großen Distanzen auffällig wird kann man sich leicht denken, wenn man sich vorstellt „wie die Quanten fliegen“, auch durch gekrümmten Raum.“
    Das hätte ich gern näher erklärt – mit meinem Wissen über Gravitation und Quantenfeldtheorie kann ich mir das nicht nur nicht leicht, sondern eigentlich gar nicht denken.

    „Es gibt ja eine Menge an Planeten, Staub und usw. die einfach nicht leuchten und daher schwer einzuschätzen sind – aber ob man das dann noch DM nennt?“
    Soweit ich es verstehe, ist das ja durch die Beobachtungen ausgeschlossen (auch Staub leuchtet thermisch.)

    „Zu den Neutrinos bin ich sozusagen neutral, das motiviert mich nicht um dafür oder dagegen zu argumentieren. Was man findet, das hat man gefunden, der Rest bleibt Spekulation.“
    Diese Aussage verstehe ich nicht. Gibt es nun Ihrer Ansicht nach Neutrinos oder nicht?

  68. @MartinB

    „Es gibt ja eine Menge an Planeten, Staub und usw. die einfach nicht leuchten und daher schwer einzuschätzen sind – aber ob man das dann noch DM nennt?“

    Soweit ich es verstehe, ist das ja durch die Beobachtungen ausgeschlossen (auch Staub leuchtet thermisch.)

    Richtig, Staub und Gas kann man direkt beobachten (Radiostrahlung, Absorption, Extinktion). Massive Objekte z.b. im Milchstraßenhalo (MAssive Compact Halo Objects, MACHOs), also etwa frei fliegende Planeten, Schwarze Löcher oder ausgekühlte Sternenreste, wurden als Kandidaten für die DM anfangs vorgeschlagen, aber Beobachtungsprogramme, die ihre Existenz durch Gravitationslinseneffekte (beim zufälligen Vorbeiziehen vor Fixsternen) nachweisen sollten, gingen wietgehend leer aus (viel zu wenige Ereignisse, um die fehlende Masse zu erklären).

    Außerdem verbietet die Urknalltheorie die Existenz einer so großen Menge Materie. Und genau diese Theorie sagt das Verhältnis der im Urknall entstandenen Elemente von Wasserstoff bis Lithium (die sich um viele Größenordnungen unterscheiden) exakt so voraus, wie sie beobachtet wird. Demnach muss die DM nicht-baryonisch sein, also kann sie nicht aus MACHOs oder Gas/Staub bestehen.

  69. @Alderamin:
    „…dann muss ja ein anderes Abstandsgesetz als 1/r² gelten, und wenn sich das auf Galaxiendurchmessern von ein paar tausend LJ auswirkt, dann würde man es doch erst Recht auf Abständen von 100 MLJ bemerken müssen“

    Nicht, wenn die modifizierte Funktion zB. so aussähe:

    (1 + a·r·e^(-b·r)) /r² .

    Mit geeigneten a und b für ein Zähler-Maximum der Funktion von ~100 könnte dadurch der ART-Grenzfall wieder für große Entfernungen r erzeugt werden.
    DM würde sich dann nur im von a und b bestimmten Bereich für r bemerkbar machen.

  70. @SCHWAR_A

    Klar kann man immer irgendeine mathematische Funktion basteln, die jede gewünschte Entfernungsabhängigkeit liefert, aber wie willst Du die physikalisch motivieren?

    Solche Potenzialverläufe kommen ja typischerweise genau dann raus, wenn man die Summe mehrerer Effekte betrachtet (z.B. den Potenzialhügel, der durch die elektromagnetische Abstoßung von Protonen bzw. ihre Anziehung durch die starke Kernkraft in Summe entsteht). D.h. eine solche Formel postuliert implizit eine neue Grundkraft. Ist das nicht weiter aus dem Fenster gelehnt als die DM-Theorie?

  71. @Alderamin:
    „…eine solche Formel postuliert implizit eine neue Grundkraft.“

    …oder aber die Kraft, die wir bisher (zu grob) erfaßten ist tatsächlich bereits eine derartige Überlagerung und wir merken das erst, wenn wir damit weitere Strecken überbrücken lassen, bzw. höher aufgelöst messen. Beispiele für sowas gibt es mehrere: Neutrino-Oszillation, Feinstrukturen in Spektren…

  72. @MartinB: Wenn man davon ausgeht dass sich Gravitonen ähnlich wie Licht verhalten, dann gilt das Geschwindigkeitslimit, und auch die Ablenkung im gekrümmten Raum. Beides wirkt sich bei großen Distanzen (und großen Massen sowie schneller Bewegung dieser) stärker aus als bei kleinen. Also ich finde das leicht.

    Das besonders lustige daran ist, weil die Gravitonen sozusagen den Raum krümmen, und durch die Raumkrümmung auch abgelenkt werden, beeinflussen sie sich gegenseitig. Das kann man nicht in einem Kraftvektor abstrahieren, man kann es nur verstehen und berechnen wenn man die Gravitonen explizit in das Modell aufnimmt.

    Zu den Neutrinos: ich finde nicht dass ich zu allem eine Meinung haben muss. Ich könnte schon irgendwas sagen, aber wenn es mir eigentlich egal ist dann ist genau das die relevante Information, nicht?

  73. @realistischer
    „dann gilt das Geschwindigkeitslimit, und auch die Ablenkung im gekrümmten Raum.“
    Huh? Das gilt doch für die Gravitation bzw. Gravitationswellen in derAllgemeinen RT genauso, ganz ohne Quanten und ist absolut kein Quanteneffekt.

    „Das besonders lustige daran ist, weil die Gravitonen sozusagen den Raum krümmen, und durch die Raumkrümmung auch abgelenkt werden, beeinflussen sie sich gegenseitig“
    Auch dieser Effekt ist bereits in der Einstein-Gleichung enthalten, der G_munu-Tensor ist nichtlinear in der Metrik:
    https://en.wikipedia.org/wiki/Einstein_tensor
    (Siehe die Gleichungen unter „explicit form“)

    „Das kann man nicht in einem Kraftvektor abstrahieren, man kann es nur verstehen und berechnen wenn man die Gravitonen explizit in das Modell aufnimmt.“
    Nein, das ist bereits in der ART im Einstein-Tensor enthalten.

    Vielleicht solltest du dir mal ein Buch über die ART besorgen, um zu sehen, was die theorie eigentlich aussagt, bevor du irgendwelche Quanteneffekte postulierst.

    „ich finde nicht dass ich zu allem eine Meinung haben muss. I“
    Nein, musst du nicht. Aber wenn du hier gegen die DM protestierst, welchen genauen Unterschied siehst du zwischen DM (bei der du ja anscheinend eine Meinung hast) und Neutrinos (zu denen du keine hast) und warum siehst du diesen Unterschied?

  74. @MartinB: Sie sagen also dass die von mir genannten Quanteneffekte schon in der ART enthalten sind. Aber woher wollen Sie das wissen? Dazu bräuchten Sie doch eine Quantengravitationstheorie, um zeigen zu können dass diese Effekte auch tatsächlich enthalten sind – und das nicht nur in Spezialfällen. Die haben Sie aber nicht, statt dessen behaupten Sie einfach dass es so wäre.

    Und ich soll’s glauben, oder wie?

    Was mein spezielles Interesse an der DM betrifft, es hat damit zu tun (Vorsicht, nur eine Äquivalenz): Die Religion hat den Teufel erfunden und alles schlechte an der Schöpfung auf ihn abgeschoben, um das gute Image ihres Gottes zu retten. Die Finanzbranche hat Bad Banks erfunden, um ihre Gewinne zu retten. Und die Physik hat die DM erfunden, um die universale Gültigkeit ihrer ART zu retten.

    Dunkle Gestalten, Schatten, überall.

    Heutzutage wird viel zu viel gerettet was eigentlich kaputt ist und durch was neues, besseres ersetzt gehörte. Nur trauen müsste man sich, das gewohnte, bequeme, sichere alte loslassen, riskieren dass in der neuen Welt, wo alle wieder von vorne anfangen müssen, plötzlich andere mehr Talent haben…

    Deutlich mehr Psychologie als Physik, hier.

  75. „Sie sagen also dass die von mir genannten Quanteneffekte schon in der ART enthalten sind. “
    Sie haben oben als Beispiel für Quanteneffekte aufgeführt:
    „Wenn man davon ausgeht dass sich Gravitonen ähnlich wie Licht verhalten, dann gilt das Geschwindigkeitslimit, und auch die Ablenkung im gekrümmten Raum. Beides wirkt sich bei großen Distanzen (und großen Massen sowie schneller Bewegung dieser) stärker aus als bei kleinen. Also ich finde das leicht.“
    ich sage, dass diese Effekte („Gravitation beeinflusst Gravitation“ und „Die Ausbreitungsgeschwindigkeit der Gravitation ist endlich“) bereits in der ART enthalten sind. Das weiß ich, weil ich ne Handvoll Bücher durchgeackert habe.

    „statt dessen behaupten Sie einfach dass es so wäre.“
    Nein, ich behaupte das nicht bloß, sondern ich habe das auch – für den fall der Selbstwechselwirkung der Gravitation – belegt. Die oben angegebene Wikipediaseite angucken und dann ein bisschen lesen, wie Gravitonen definiert sind (die Tatsache, dass es keine funktionierende QG gibt bedeutet nämlich nicht, dass wir gar nichts über Gravitonen wissen), dann können Sie das selbst nachrechnen.

    Für die endliche Ausbreitungsgeschwindigkeit gilt dasselbe. Das sind keine leeren Behauptungen, sondern steckt direkt in der Formeln der ART.

    „Und die Physik hat die DM erfunden, um die universale Gültigkeit ihrer ART zu retten.“
    So wie sie 1932 das Neutrino „erfunden“ hat, um die Impuls- und Energieerhaltung zu retten. Es war etwas, für das es zunächst keinen Nachweis gab, das aber notwendig erschien. Wir wissen, dass sich Galaxien anders verhalten, als sie es auf grund ihrer beobachtbaren Masse tun müssten. Bisher ist es niemandem gelungen, eine Modifikation der ART (wie MOND) vorzuschlagen, die die Beobachtungen (siehe den Post von Alderamin) so gut erklären kann wie die Annahme, dass es DM gibt.

    Damit ist die DM momentan das beste Modell zur Erklärung der Beobachtungen. Das heißt nicht, dass es stimmen muss – aber es heißt, dass jeder, der die DM ablehnt, in der Pflicht ist, eine alternative Theorie vorzulegen, die die Beobachtungen ähnlich gut erklärt.

    In der Physik reicht es nicht, zu sagen, was man gern aus bestehenden Theorien wegwerfen möchte – man muss auch etwas Konkretes vorschlagen, was dies ersetzen soll. Siehe auch hier:
    https://www.scienceblogs.de/hier-wohnen-drachen/2011/08/buchersommer-r-feynman-the-character-of-physical-law-wie-man-neue-gesetze-findet.php

    „Deutlich mehr Psychologie als Physik, hier.“
    Solange Sie das „hier“ auf ihren eigenen Standpunkt beziehen, sicher.

  76. @MartinB: Sie haben Ihr Wissen also aus Büchern. Gut. Und woher haben die Autoren dieser Bücher ihr Wissen? Eine Quantengravitationstheorie haben die nämlich auch nicht, denen fehlt genauso die Beweismöglichkeit für die Aussagen die Sie denen geglaubt und hier wiedergegeben haben. Nur ich glaube das eben nicht mehr, ich verlange Beweise! SIe und die anderen hier sind die, die diesbezüglich in der Beweispflicht stehen, nicht ich!

    Und ja, das Aufdecken von Vorurteilen hat viel mit Psychologie zu tun. Und es ist wichtig!

  77. „ch verlange Beweise!“
    Schön. Da sind Sie in der Wissenschaft aber falsch, Beweise haben wir nicht und so funktioniert Wissenschaft auch nicht.

    Wir stellen Theorien auf und überprüfen sie an Hand von Experimenten oder Beobachtungen. Wenn diese die Theorie nicht widerlegen, dann akzeptieren wir die Theorie (vorläufig) als korrekt.

    Die ART hat bisher alle Tests gut bestanden und wird deswegen akzeptiert. Mit unserem Wissen über Quantenfeldtheorien können wir daraus bereits einiges über die Eigenschaften von Gravitonen ableiten, auch wenn es noch keine wirklich funktionierende Quantengravitation gibt.

    Die von Ihnen als mögliche Quanteneffekte angeführten Effekte (Selbstwechselwirkung, endliche Geschwindigkeit) sind keine Quanteneffekte, sondern bereits Bestandteil der ART selbst (und werden damit auch Bestandteil jeder Quantengravitation sein müssen, sonst wäre die mit Beobachtungen nicht konsistent.)

    „SIe und die anderen hier sind die, die diesbezüglich in der Beweispflicht stehen, nicht ich!“
    Nein. So funktioniert Wissenschaft nicht (siehe oben und auch den Link, den ich gestern gepostet habe). Es nützt niemandem, wenn Sie sagen „Eure Theorie ist mir nicht gut genug, ich verlange eine Quantengravitation“.
    Hätten wir ja gern, haben wir aber (noch?) nicht.
    Trotzdem kann man aber einige Aussagen darüber machen, welche Eigenschaft eine Quantengravitation haben muss, um mit Beobachtungen konsistent zu sein.

    Bezüglich der Dunklen Materie ist es bisher niemandem gelungen, eine Modifikation der ART zu finden, die die Beobachtungen erklärt ohne DM anzunehmen. (Die Alternativ-Theorie MOND ist wohl nicht mehr im Rennen.) Solange wird die Existenz der DM als beste mögliche Erklärung akzeptiert (was aber nicht heißt, dass nicht Leute nach Alternativen suchen).

    Einfach zu sagen „DM finde ich doof, das geht bestimmt anders“ ist keine Wissenschaft.

  78. @MartinB: Bezüglich der Aussage, dass die ART schon diese Effekte beinhaltet die sich aus einem Quantengravitationsverhalten ergeben, sind Sie sehr wohl in der Beweispflicht. Sie haben’s ja behauptet. Aber Sie können derzeit garnicht zeigen dass ART und Quantengravitation äquivalent sind, weil derzeit nur eins von beiden verfügbar ist. Also entweder Sie nehmen die Aussagen zurück, oder Sie bleiben weiterhin in der Beweispflicht.

    Und wenn es Sie noch so stört, aber genau so funktioniert Wissenschaft: indem man keine Ausflüchte hinnimmt sondern Beweise einfordert!

  79. „Aber Sie können derzeit garnicht zeigen dass ART und Quantengravitation äquivalent sind, weil derzeit nur eins von beiden verfügbar ist. “
    Das ist wie mehrfach gesagt falsch.
    Auch wenn wir nicht alles über die QG wissen, wissen wir doch, dass sie im klassischen Grenzfall wieder zur ART führen muss. Daraus kann man einige Folgerungen über die Eigenschaften von Gravitonen ableiten, z.B. die, dass diese eine Selbstwechselwirkung haben müssen, weil der Einsteintensor in der ART nicht-linear ist.

    (Für schwache Gravitationsfelder nähern Sie den Metrischen Tensor als g_munu mit 1+h_munu an und quantisieren dann h_munu mit den üblichen Mitteln der Quantenfeldtheorie. Dann sehen Sie sofort, dass es eine Selbstwechselwirkung von Gravitonen geben muss – es sei denn, der ganze Ansatz ist falsch und es gibt gar keine Gravitonen, weil die ART sich nicht in dieser Weise quantisieren lässt. Das ist aber ja nicht Ihr Standpunkt, da Sie ja oben selbst die Gravitonen ins Spiel gebracht haben.)

    Dass Sie das nicht verstehen, heißt nicht, dass es falsch ist, sondern nur, dass Sie es nicht verstehen.

    „so funktioniert Wissenschaft: indem man keine Ausflüchte hinnimmt sondern Beweise einfordert!“
    Dafür, dass Wissenschaft „Beweise einfordert“ hätte ich gern einen Beleg. Beweise gibt’s in der Mathematik, in der Wissenschaft gibt’s Belege, mehr nicht.

  80. @MartinB: Also wenn die ART und die QG so äquivalent sind wie Newton und ART, dann würde das voll und ganz meinen Erwartungen entsprechen.

    Aber ob Beweis oder Beleg, es scheitert nicht an meinem Terminologiefehler sondern daran dass Sie weder das eine noch das andere liefern können.

  81. „sondern daran dass Sie weder das eine noch das andere liefern können.“
    Siehe oben – dass Sie meine Ausführungen nicht verstehen, heißt nicht, dass sie falsch sind.

  82. „Für schwache Gravitationsfelder nähern Sie den Metrischen Tensor als g_µν mit 1+h_µν an und quantisieren dann h_µν mit den üblichen Mitteln der Quantenfeldtheorie. Dann sehen Sie sofort…“

    An solchen Sätzen liegt es wohl, daß „(lay)man“ sich schwer tut, darin auch nur den Ansatz eines Belegs zu sehen. Das ist eine Art kompakter Geheimsprache, die man erst lernen muß… Hierzu einen Laien-Zugang zu schaffen wäre vielleicht mal ein Blog-Artikel wert, wahrscheinlich aber eher mehrere…;-)

  83. @SCHWAR_A
    „An solchen Sätzen liegt es wohl, daß „(lay)man“ sich schwer tut, darin auch nur den Ansatz eines Belegs zu sehen. “
    Ich schreibe solche Sätze ja auch normalerweise nicht. Wenn aber jemand ankommt und behauptet, die Physiker hätten ja alle keine Ahnung etc., dann muss man vielleicht schon mal klarmachen, dass es mit ein bisschen herumfantasieren (also ich könnte mir vorstellen, dass Quanten da irgendwie wichtig sind) nicht getan ist.

    „Hierzu einen Laien-Zugang zu schaffen wäre vielleicht mal ein Blog-Artikel wert, wahrscheinlich aber eher mehrere.“
    Ein paar Stichworte habe ich schon, irgendwann wird die QFT-Serie da ankommen (im Moment grüble ich noch über Ein-Photon-Zustände und kohärente Zustände, da hakt meine Anschauung).

  84. @MartinB:
    „im Moment grüble ich noch…“

    Mach‘ doch ein Zwischen-Brainstorming-Thema darüber auf… Ich denke, Bjoern und Niels könnten ’ne gute Hilfe werden… und mich interessiert der Werdegang einer Idee auch.

    Ein präsentiertes fertiges Ergebnis versteckt ja allzu oft die dahinführenden Gedankengänge, auch die Irrwege und Fallstricke, die ich jedoch immer als recht hilfreich empfinde.

  85. @MartinB: Aha, also Sie grübeln noch über Ein(!)-Photon-Details, aber dass eine QG, wenn sie fertig ist, ganz genau die ART ergeben wird, das wissen Sie schon ganz sicher…

    Bei so viel offensichtlicher Überheblichkeit ist klar dass Sie jede Beweis- oder Beleg-Forderung ablehnen müssen. Mit dem „die dummen anderen verstehen nix“-Blabla geht das auch ganz bequem.

  86. @Realistischer

    Die ART beschreibt das, was durch Beobachtung gesichert ist. Wenn eine QG der ART im durch Beobachtung gesicherten Bereich widerspricht, widerspricht sie den Beobachtungen und damit der Realität. Das ist nicht überheblich sondern trivial!

    Nichts anderes hat Martin gesagt. Auch wenn wir eine neue Theorie der Gravitation entwickeln werden, fallen deswegen in Zukunft nicht Steine nach oben.

    Wer mitreden will, muss sich leider in die Materie einarbeiten, auch wenn’s weh tut, denn ART ist ein bisschen kompliziert, ansonsten: hier.

  87. „Wer mitreden will, muss sich leider in die Materie einarbeiten, auch wenn’s weh tut, denn ART ist ein bisschen kompliziert, ansonsten: hier.“

    Nein nein und nochmal nein wenn ich sage Steine fliegen nach ob dann ist das so , oder ich steh aufem Kopf……

    Hmh ob Realistischer wohl Ingenieur ist ?

  88. @MartinB
    „Für die endliche Ausbreitungsgeschwindigkeit gilt dasselbe. Das sind keine leeren Behauptungen, sondern steckt direkt in der Formeln der ART.“

    Das stimmt. Und gleichzeitig steckt noch eine unendliche Ausbreitungsgeschwindigkeit in den Formeln der ART.

    Aus diesem Grund funktioniert die ART auch so schön in unserem Sonnensystem, bei welchem die ART formelmäßig in den Newtonschen Gernzwert übergeht. Sämtliche typischen ART Terme werden dann wegen „zu kleiner“ Geschwindigkeitem und „zu kleiner“ Massen praktisch 0 bzw. liefern nur Beiträge weit unterhalb des Meßwertrauschens.

    Einzig die unendlich schnelle Wirkungsgeschwindigkeit Newtons ist dann in den Formeln noch enthalten. Da jedoch der die Wirkungsgeschwindigkeit enthaltende Term 1/oo = 0 ist, wird er auch bei Newton nicht angegeben, da er die Bedeutung „….. + 0“ hat. Kann man sich also ersparen.

    Tatsächlich müßte jedoch der die endliche Wirkungsgeschwindigkeit enthaltende Term noch lauten:
    …+ G*m*v/(r²c)

    Es müßte daher die DGL für die Beschleunigung im sogenannten Grenzfall lauten:

    b = – G*m/r³ * R + G*m*v/(r²c) + Einsteinterm/c²

    Der Einstermterm (enthält Gravitationspotentiale und Geschwindigkeiten) hat wegen niedriger Geschwindigkeit und kleiner Potentiale den Wert ca. 0.

    Da jedoch dieser notwendige Term „G*m*v/(r²c)“ fehlt, kann dies nur darauf zurückgeführt werden, daß hierbei mit einem c=oo gerechnet wird, sodaß G*m*v/(r²c) = 0 ist.

    Damit ist bewiesen, daß die ART mit 2 unterschiedlichen Wirkungsgeschwindigkeiten arbeitet und sich daher selbst widerspricht. Newton hat sich hervorragend bestätigt, wenn man die „üblichen“ störenden Gegebenheiten im Sonnensystem ebenfalls noch berücksichtigt.

    Damit widerlegt die ART auch gleichzeitig die SRT, welche ebenfalls c als wirkende Geschwindigkeit behauptet. Ebenso gut könnte man die Schallgeschwindigkeit der Luft als Wirkgeschwindigkeit behaupten.

    Rechnet man mit der korrigierten ART Formel z.B. den Mondumlauf aus, stellt man fest, daß sich der Mond je Umlauf um ca. 400 m von der Erde entfernen müßte. Das ist jedoch noch nicht festgestellt worden.

    Der Mond funktioniert schlicht nach:

    b = – G*m/r³ * R

    und eben nicht nach:

    b = – G*m/r³ * R + G*m*v/(r²c)

  89. Die ART ist aber nicht durch alle(!) Beobachtungen gesichtert, denn sonst bräuchte man keine DM!

    Um das nochmals klarzustellen: die Newtonschen Gravitationsgesetze sind auch durch Beobachtungen gesichert gewesen, aber irgendwann ist man an die Grenzen ihrer Gültigkeit gestossen. Dasselbe kann auch mit der ART passieren – und ich meine, es ist schon passiert.

    Ob es wirklich schon passiert ist oder nicht, darüber kann man gut streiten. Und in diesem Zusammenhang lasse ich mich auch nicht von den Verweisen auf die schön komplexe ART beeindrucken, denn die offene Frage ist doch warum damit nicht alle Beobachtungen erklärt werden können.

  90. @ Realistischer

    „Um das nochmals klarzustellen: die Newtonschen Gravitationsgesetze sind auch durch Beobachtungen gesichert gewesen, aber irgendwann ist man an die Grenzen ihrer Gültigkeit gestossen. “

    Das ist eine reine Behauptung! Sie resultiert nur aus der Unkenntnis der wahren Verhältnisse. So hat man z.B. bei den Galaxien und den Umlaufgeschwindigkeiten ein Keplermodell unterstellt, wo also die Umlaufgeschwindiigkeiten nach außen hin stark abfallen müßten. Das ist ein unglaublicher Blödsinn, das darf man eben nicht.

    Wenn man irgendeine Scheibe mit irgendeiner „plausiblen“ Massendichte abhängig vom Zentrum rechnet, ergeben sich „sofort“ die beobachteten Umlaufgeschwindigkeiten auch ohne DM!

    Es ist kein Grund vorhanden, für Newton irgendwelche Gültigkeitsgrenzen annehmen zu müssen.

  91. @neulich

    Ja, genau, die ART ist wieder mal falsch, und wenn man ihre Formeln „korrigiert“, macht sie auch noch falsche Vorhersagen…

    Wenn man sie aber nicht korrigiert, sagt sie Effekte wie Zeitdilatation, Ablenkung von Licht im Schwerefeld, Frame Dragging, Periheldrehung, Graviationswellen etc. voraus, die alle durch Messungen nachgewiesen wurden.

    Das, was Du als unendliche Ausbreitungsgeschwindigkeit deutest, ist aber gar keine Ausbreitung. Eine Masse krümmt um sich den Raum und nach der SRT spielt es dabei keine Rolle, ob sie in Ruhe oder in gleichförmiger Bewegung ist (hängt ohnehin vom Beobachter ab). Im gekrümmten Raum um die Masse bewegen sich andere Körper auf Geodäten, die aufgrund der Raumkrümmung keine geraden Linien mehr, sondern Kegelschnitte sind. Da braucht sich nichts auszubreiten. Der gekrümmte Raum umgibt die Masse statisch und die Gravitation ist nichts als eine Scheinkraft, denn ein Probekörper folgt einfach im freien Fall seiner Geodäte, wie er das im feldfreien Raum, wo diese eine Gerade ist, auch tun würde. Nur wenn man ihn daran hindert (wie das z.B. der feste Boden unter Deinen Füßen bei Dir tut), spürt man eine Kraft.

    Nur Änderungen im Schwerefeld aufgrund von beschleunigter Bewegung oder Massenverlust führen zu Änderungen des Schwerefelds, die sich in Form von Gravitationswellen ausbreiten, und die haben Lichtgeschwindigkeit.

  92. @Alderamin

    „Wenn man sie aber nicht korrigiert, sagt sie Effekte wie Zeitdilatation, Ablenkung von Licht im Schwerefeld, Frame Dragging, Periheldrehung, Graviationswellen etc. voraus, die alle durch Messungen nachgewiesen wurden.“

    Die Zeitdilatation ist vollständig widerlegt. Ablenkung von Licht im Schwerefeld gibt es auch bei Newton. Framedragging ist total erlogen, wie man seit Gravity Probe B weiß. Periheldrehung resultiert aus einem vernachlässigten Quadrupolmoment der Sonne und weiterer unberücksichtigt gebliebener Effekte. Gravitationswellen hat man nicht gefunden.

    Obige Formeln beweisen die Falschheit der ART.

  93. @Realistischer

    Ich versteh nicht ganz, warum du das Korrespondenzprinzip zwischen Newton -> ART -> QG nicht verstehen/anerkennen willst.
    Du schreibst ja schon selbst:

    Um das nochmals klarzustellen: die Newtonschen Gravitationsgesetze sind auch durch Beobachtungen gesichert gewesen, aber irgendwann ist man an die Grenzen ihrer Gültigkeit gestossen

    Das Newtonsche Gravitationsgesetz ist durch Beobachtungen gut gesichert und die ART, als Theorie mit größerem Gültigkeitsbereich, muss primär für den Gültigkeitsbereich von Newton die selben Ergebnisse liefern, da sie sonst mit den vorhandenen Beobachtungen nicht übereinstimmen und sich damit als Erweiterung schon disqualifizieren würde.

    Stimmst du mir da soweit zu? Denn jetzt kommt der Korrespondenz-Trick:

    Tausche in meinem obrigen Satz „Newtonsche Gravitationsgesetz“ mit „ART“ und „ART“ mit „QG“ aus. Fällt dir was auf? Beide Sätze müssen zwingend richtig sein und daraus folgt MartinB auch, dass die ART in der QG behinhaltet sein muss, in welcher Form auch immer.

  94. @Realistischer
    „Dasselbe kann auch mit der ART passieren – und ich meine, es ist schon passiert.“
    Dasselbe *kann* der ART in der Tat passieren – aber die Dinge, die bereits bestätigt sind, werden dadurch nicht falsch. Eine Erweiterung der ART müsste in den Bereichen, in denen die ART bereits experimentell gut abgesichert ist, eben dieselben Ergebnisse liefern.
    Und das gilt auch, wenn diese Erweiterung eine QG ist – auch die muss in den Fällen, die wir schon verstehen, das richtige ergebnis liefern, z.B. dafür:
    https://en.wikipedia.org/wiki/PSR_B1913_plus_16
    Und das setzt den Möglichkeiten, wie sich Gravitonen verhalten können, eben gewisse Grenzen.

  95. @neulich

    Die Zeitdilatation ist vollständig widerlegt.

    Klar: https://hyperphysics.phy-astr.gsu.edu/hbase/relativ/airtim.html

    Ablenkung von Licht im Schwerefeld gibt es auch bei Newton.

    Nur kommt bei Einstein der doppelte Wert heraus, und der wurde bei der SoFi 1919 auch bestätigt…

    Framedragging ist total erlogen, wie man seit Gravity Probe B weiß.

    So, so: https://en.wikipedia.org/wiki/Gravity_Probe_B

    Periheldrehung resultiert aus einem vernachlässigten Quadrupolmoment der Sonne und weiterer unberücksichtigt gebliebener Effekte.

    Wie schafft es ein so gleichförmig kugelförmiger Körper wie die Sonne, ein Quadrupolmoment zu haben?

    Gravitationswellen hat man nicht gefunden.

    Oder vielleicht doch? https://de.wikipedia.org/wiki/PSR_1913%2B16

    Offenbar besonders schwerer Fall von Dunning-Kruger und damit erübrigt sich auch jede weitere Diskussion…

  96. @MartinB

    Egal wer oder ob, Tatsache ist, daß du dir höchstens um Inkarnationen Gedanken machst. Du bist also ein typischer Esoteriker.

    So vermeidest du, dich mit Inhalten auseinandersetzen zu müssen. Die kannst du jedoch nicht widerlegen.

    @Alderamin

    Bereits daß es überhaupt ein Experiment zur Zeitdilatation gegeben hat, widerlegt die Zeitdilatation!
    Abgesehen davon wurden während des „Experiments“ die Uhren nachgestellt. Beobachtet wurde dagegen nicht, daß die Turbinen rückwärts liefen, als das Flugzeug aus 2 Tagen Zukunft zurück in die Vergangenheit eintauchte. Auch schlugen die Herzen der Piloten beim Landeanflug ebenfalls nicht rückwärts.

    Das ist nachgewiesen!

    Lichtablenkung:
    Es wurde mehr festgestellt, als Einstein „erlaubte“. Die Atmosphäre um die Sonne ist eben „lufthaltig“ und hat daher auch einen optischen Brechungsindex. Man kann leicht ausrechnen, wie sich eine „Luftdichte“ verursacht durch den Sonnenwind von ca. 1e-6 bis 1e-7 kg/m³ in näherer Sonnenumgebung (10 * r ) auswirkt.

    Framedragging:
    Der primäre Effekt war etwa 100 fach größer als erwartet. Durch entsprechende „Fehlerrechnung“ hat man dann die höchste Präzision einsteinbestätigend hinbekommen. Man muß schon sehr blöde sein, um den Schwindel nicht durchschauen zu können.

    Außerdem waren die Präzisionskugeln sowieso eher Eier.

    kugelförmiger Körper wie die Sonne:

    Die Sonne ist keine Kugel. Sie rotiert und innerhalb ist sie auch recht turbulent. Über die Massenverteilung im Inneren ist nichts bekannt. Es ist daher alleine aufgrund der Rotation unmöglich, daß sie kein Quadrupolmoment haben kann.

    Gravitationswellen:
    Diese Effekte sind nur dadurch bedingt, daß ein 3. Himmelskörper mit im Spiel ist, um welchen das Pärchen kreist.

    Brauchst nicht zu diskutieren. Du kannst es nicht!

  97. @lordhoengy, MartinB: dass die ART in der QG enthalten sein sollte, genau davon gehe ich auch aus. Aber so wie ich MartinB verstanden habe, meint er das umgekehrte, nämlich dass die QG in der ART enthalten ist, weil ja schon alles in deren Feldgleichungen enthalten ist und man daher keine Gravitonen braucht — und dem kann ich einfach nicht zustimmen.

  98. @Realistischer
    Dann hast du mich nicht verstanden. Die QG muss *im klassischen Grenzfall* die ART ergeben, und daraus kann man bereits einiges über das Verhalten von Gravitonen ableiten.

  99. @MartinB
    „Jedenfalls glänzt neulich gerade bei Thilo durch andere Scheußlichkeiten:“

    Wenn du 2 Zoll große und 8 Zoll dicke Menschen als Scheußlichkeiten bezeichnest, könntest du ein Problem bekommen. Noch dazu, wo diese Menschenrasse nachweislich von den Nazis ausgerottet wurde!

    Oder willst du das etwa auch leugnen?

  100. @Realistischer

    Da wirst du (IMHO) MartinB falsch verstanden haben. Das, was er sagen wollte (gesagt hat), ist dass man zur Simulation einer Gravitation mit endlicher Geschwindigkeit (+ Selbstwechselwirkung) keine QG im Sinne von Gravitonen braucht, da diese beiden Prinzipien bereits in der ART enthalten sind, siehe bspw. Abberation .

  101. @neulich: „Noch dazu, wo diese Menschenrasse nachweislich von den Nazis ausgerottet wurde! Oder willst du das etwa auch leugnen? „

    Worum auch immer es hier geht – es hat mit dem Thema nichts zu tun. Also lasst das.

  102. @MartinB: Scheint ein Missverständnis gewesen zu sein, oder auch nicht. Dass die ART in der QG enthalten ist – aber nicht die QG in der ART – impliziert dass per QG etwas erklärbar sein wird das per ART nicht erklärbar ist — und das kann dann sehr wohl die DM betreffen. Und dass dies so werden wird finde ich ganz und gar nicht unwahrscheinlich, sondern im Gegenteil höchst wahrscheinlich.

  103. @Florian Freistetter

    Es war nicht ich, der das Thema in eine andere Richtung lenkte! Sondern
    MartinB· 18.04.12 · 15:21 Uhr

    Offenkundig wußte er nichts mehr zu entgegnen und glaubte daher, mit Inkarnationen punkten zu können. Das sind dann immer die letzten Rettungsanker von Esoterikern.

  104. Wer bezahlt mir jetzt ne neue Tischplatte ? Vielleicht sollte neulich über seine Kommentare “ Lesen auf eigene Gefahr “ schreiben…….

  105. „und das kann dann sehr wohl die DM betreffen.“
    Womit wir wieder am Anfang wären, nämlich der Frage, wie ein Quanteneffekt auf einer Skala von einigen Tausend Lichtjahren wirken soll, aber nicht auf einer von einigen Kilometern oder einigen Millionen Kilometern.

    Deswegen war ja ganz oben meine Frage
    „mit meinem Wissen über Gravitation und Quantenfeldtheorie kann ich mir das nicht nur nicht leicht, sondern eigentlich gar nicht denken.“
    Und dann kamen Sie mit der Selbstwechselwirkung und der endlichen Ausbreitungsgeschwindigkeit, was beides Effekte sind, die eben schon in der ART selbst enthalten sind.

    Womit Sie immer noch eine Erklärung schuldig sind, welcher Quanteneffekt nun wie zu den Effekten führen soll, die man zur Zeit mit Dm erklärt.

  106. Da werden einem all die schönen Wörter verhunzt… Gemäß dem thread ist ’neulich realistisch‘ gleichbedeutend mit ‚ewiggestrig surreal‘. Weia^^

  107. @MartinB: Ich bezweifle aber stark, dass diese Effekte in ihrer allgemeinen Form in der ART enthalten sind. Ein Gewusel an Gravitonen lässt sich nicht ohne substanziellen Informationsverlust auf einen Tensor oder zwei reduzieren, das geht nur in Spezialfällen.

    Aber um das objektiv zeigen oder widerlegen zu können braucht man die QG, die man aber noch nicht hat. Weshalb ich Ihre Aussage ablehne, dass doch eh schon alles in der ART enthalten sei – Sie können das nämlich auch nicht zeigen. Womit wir wieder am Anfang wären…

  108. @Realistischer
    „Ein Gewusel an Gravitonen lässt sich nicht ohne substanziellen Informationsverlust auf einen Tensor oder zwei reduzieren, das geht nur in Spezialfällen.“
    Ja, da ist was dran. Das kennt man auch z:B. aus der Quantenelektrodynamik. Das gewusel aus Photonen lässt sich nur in einem Spezialfall auf elektromagnetische Felder reduzieren. Dieser Spezialfall heißt „klassischer Grenzfall“ und liefert die Maxwell-Gleichungen.

    Wenn Sie behaupten, dass das bei der QG anders sein soll und dass diese auf der Längenskala von Galaxien nicht die bekannte ART, sondern etwas anderes liefern soll, dann brauchen Sie dafür ein Argument, anonsten ist es Wunschdenken – im Sonnensystem gilt die klassische ART, aber auf Galaxienebene schlagen dann plötzlich wieder Quanteneffekte zu? Wie soll das gehen?

  109. @MartinB: Ja, wie soll das gehen? Die QG soll uns das beantworten.

    Sicher ist jedenfalls das im Sonnensystem die Gravitation von der Sonne dominiert wird, während es in einer Galaxie kein solches dominierendes Zentrum gibt. Das wäre mein erster Kandidat für einen Gültikgeitsunterschied, denn per dominierender Zentralmasse erhält man noch kein richtig tolles Gewusel an Gravitonen.

    Obwohl, in Übergangsbereichen schon, an denen könnte man womöglich gut forschen. Die Lagrange-Punkte fallen mir ein, wo es keine Kraft aber trotzdem Gravitonen geben müsste, also ein chaotisches Hin- und Herspringen das aber in Summe nicht zu einer Ortsveränderung führt. Und je weiter weg vom Gleichgewichtspunkt, desto deutlicher würde die Netto-Ortsveränderung ausfallen.

    Somit hätten wir sogar schon einen Experiment-Ansatz. Könnte man sowas heutzutage schon messen, Gravitationswirkung auf Quantenniveau?

  110. MartinB·
    „Es gibt ja eine Menge an Planeten, Staub und usw. die einfach nicht leuchten und daher schwer einzuschätzen sind – aber ob man das dann noch DM nennt?“
    Soweit ich es verstehe, ist das ja durch die Beobachtungen ausgeschlossen (auch Staub leuchtet thermisch.)

    Sie überschätzen die Möglichkeiten. Inwiefern ist ein Gegenstand aus Glas im Gefäß mit Wasser zu sehen? Genauso: Inwiefern ist ein Planet mit gleicher Temperatur wie Staub um ihn herum in der Staubwolke zu sehnen?

    Hier fügt Florian Freistetter gleich ein paar Asteroiden in den Stabring und zwei Planeten außerhalb hinzu und hier werden die Trillionnen von Kometen mit Gesamtmasse von 110 Erdmassen und ein Planet ins Spiel gesetzt. Warum kann man das bloß machen, wenn man nur Staub sieht? – Weil die Gesteinsbrocken in den Staubwolken durch nichts zu erkennen sind. Deshalb geht es ganz locker, beliebe Anzahl von den Kleinkörpern wie Asteroiden und Kometen in die Staubwolken hineinzusetzen, wenn es nötig ist.

  111. MOND, TeVeS ( und ART) sind doch „nur“ Repräsentanten bestimmter Klassen von „Skalarfeldmetriken“ –> https://arxiv.org/abs/0805.1726
    MOND selbst ist wohl wirklich zu ‚einfach‘ um tatsächlich die Beobachtungsdaten zu erklären. Aber es gibt ja noch viele andere Möglichkeiten für ‚modifizierte‘ Skalarfeld-Tensor-Metriken ( mit ART als Grenzfall). In einem vorhergehenden Kommentar wurde mal erwähnt ( im Zusammenhang zur Modifikation von 1/r²), daß in gewisser Weise Belibigkeit für das „Zusammenbasteln“ von mathematischen Funktionen gilt aber ggfs. die physikalische Begründung/Motivation fehlen würde – hat man die nicht hier eben um die Beobachtungen zu erklären/beschreiben zu können? Hier eine dementsprechend modifizierte ART-type-of-Gravitytheory zu nehmen ist doch auch nicht wesentlich mehr „deus ex machina“ als (noch) hypothetische DM-Teilchen(WIMPs) anzunehmen – auch wenn das schön mit der E_8 Lie-Gruppe paßt, doch auch da werden ja nicht alle damit impliziten potentiellen 248-Teilchen per se als existent angenommen.
    Auch wird (und wurde in einem vorherigen Kommentar) gerne „DM vs. modifizierte ART“ angemerkt – warum eigentlich? Beide Ansätze versuchen die Beobachtungen zwar alleinig zu erklären aber warum schließen sie sich gegenseitig aus? Ist nicht auch ein „teils-teils“ möglich?
    Und noch eine Frage zur „QuantenLoopGravitation(QLG)“ – ein entsprechender Satellit ist doch jetzt schon seit einiger Zeit beim Beobachten von Licht/Gammastrahlen, ist da jetzt schon ein von der QLG vorrausgesagter Geschwindigkeitsunterschied festgestellt worden oder ist das noch offen oder ist die QLG vom Tisch?

  112. @lordhoengy: Die Sonne enthält fast die gesamte Masse des Sonnensystems (>99%). Das könnte man totale Dominanz nennen.

    Zum Vergleich, wieviel Prozent der Gesamtmasse einer Galaxie entfallen auf so ein supermassives SL? 4 Mio. Sonnenmassen ist sehr viel für ein einzelnes Objekt, im Vergleich zu 100+ Mrd. Sternen kann aber von einer Dominanz des SL über die gesamte Galaxie keine Rede sein.

    Und in einem Galaxienhaufen, in dem die DM erstmals „nachgewiesen“ wurde, sind die Verhältnisse nochmals deutlich anders – da gibt’s als auffälligsten Unterschied keine Rotationssymmetrie mehr.

  113. @StefanL

    In einem vorhergehenden Kommentar wurde mal erwähnt ( im Zusammenhang zur Modifikation von 1/r²), daß in gewisser Weise Belibigkeit für das „Zusammenbasteln“ von mathematischen Funktionen gilt aber ggfs. die physikalische Begründung/Motivation fehlen würde – hat man die nicht hier eben um die Beobachtungen zu erklären/beschreiben zu können?

    Das war anders gemeint.
    Beliebig zusammengebastelte Formeln würden eine rein deskriptiv Lösung für einen Spezialfall liefern.
    In der Physik versucht man allerdings universell gültige Theorien zu finden, bei denen man Formeln aus grundlegenden Prinzipien herleitet.
    Theorien bringen somit zwangsläufig immer eine Art physikalische Begründung/Motivation für Formeln mit.

    Deswegen liefert die Annahme von DM bei Gültigkeit der ART auch nicht einfach nur eine passende Modifikation der Gravitationsformel für die Bewegung von Galaxien, sondern es ergeben sich noch zahlreiche weitere Folgerungen, etwa für die Kosmologie, Gravitationslinsen, …
    Wenn man diese Folgerungen durchrechnet, bemerkt man, dass die Vorhersagen, die bei der Annahme DM herauskommen, perfekt zu unseren Beobachtungsdaten passen.
    Rechnungen ohne dunkle Materie liefern dagegen Unsinn.
    Die Annahme dunkler Materie im Rahmen der ART ist also eine ziemlich einfache, aber mächtige Lösung, die viele verschiedene, eigentlich unabhängige Probleme löst.

    Die ganze heutige Kosmologie beruht zum Beispiel wesentlich auch auf der Annahme der Existenz von DM.
    Wie Aleramin schon erwähnt hat, reichen die mit ihrer Hilfe möglichen (zu den Beobachtungen passenden) Vorhersagen vom Verhältnis der Elemente im frühen Universum über die Galaxienentstehung bis hin zum Aussehen der größtmöglichen Strukturen im Universum.

    Auch wird (und wurde in einem vorherigen Kommentar) gerne „DM vs. modifizierte ART“ angemerkt – warum eigentlich? Beide Ansätze versuchen die Beobachtungen zwar alleinig zu erklären aber warum schließen sie sich gegenseitig aus?
    Ist nicht auch ein „teils-teils“ möglich?

    Ein teils-teils ist natürlich prinzipiell immer möglich.

    In der Physik versucht man allerdings unnötig komplizierte Lösungen zu vermeiden und mit möglichst wenigen Grundannahmen auszukommen (Stichwort Ockhams Rasiermesser, Epizykeltheorien, …).

    Hier eine dementsprechend modifizierte ART-type-of-Gravitytheory zu nehmen ist doch auch nicht wesentlich mehr „deus ex machina“ als (noch) hypothetische DM-Teilchen(WIMPs) anzunehmen

    Da kannst du als Antwort jetzt viel von den oben genannten Dingen übernehmen.

    Außerdem hat man bisher einfach noch keine passende modifizierte „ART-type-of-Gravitytheory“ gefunden, die keine größeren Probleme verursacht, aber das selbe leistet wie der Ansatz „ART einschließlich dunkle Materie“.
    Deswegen muss man aber diese Alternative momentan eigentlich nicht groß diskutieren.

    Und noch eine Frage zur „QuantenLoopGravitation(QLG)“ – ein entsprechender Satellit ist doch jetzt schon seit einiger Zeit beim Beobachten von Licht/Gammastrahlen, ist da jetzt schon ein von der QLG vorrausgesagter Geschwindigkeitsunterschied festgestellt worden oder ist das noch offen oder ist die QLG vom Tisch?

    Du meinst das Fermi Gamma-ray Space Telescope?
    Da gab es vor einiger Zeit ein nature-Paper (ist auch bei ohne Bezahlschranke bei arxiv zu finden):
    https://www.nature.com/nature/journal/v462/n7271/edsumm/e091119-06.html

    Dort war bei einem 31 GeV Photon und einer zurückgelegten Strecke von fast 12 Milliarden Lichtjahren keine Verletzung der Lorentzinvarianz festzustellen.
    Allerdings kann damit die QLG gar nicht vom Tisch sein, da die bisherige QLG gar keine definitive Aussage darüber macht, wie groß dieser Effekt mindestens sein muss.
    31 GeV ist zwar eine der höchsten Photonenenergien, die je bei einem Gammastrahlenblitz beobachtet wurden. Diese Energie könnte aber eben immer noch deutlich zu klein sein.
    Das ist wie bei den Extradimensionen. Da kann man durch Experimente auch immer nur neue Untergrenzen finden, das Ganze aber nie endgültig falsifizieren.
    Im nature-Paper hat man gefunden;
    No evidence for the violation of Lorentz invariance was found in the GRB 090510 spectrum, at least down to a limit of the Planck length divided by 1.2.
    Allerdings beruht die ganze Arbeit auf einem einzigen 31 GeV Photon!

    Allerdings hilft die Schleifenquantengravitation hier doch überhaupt nicht weiter, oder?
    Meines Wissens kommt die QLG aber momentan nämlich auch nicht ohne dunkle Materie aus es gibt auch keine Anzeichen dafür, dass sich das mal ändern wird?

  114. SCHWAR_A: https://astronews.com/news/artikel/2012/04/1204-022.shtml

    Endlich bewegt sich was in richtige Richtung! Dass unsere Umgebung einigermaßen sauber sein soll, lässt sich ganz leicht aus der Rotationskurve herauslesen. Sonne befindet sich gerade in der Mitte fallender Flanke. Diesen Bereich können wir auch als Keplerscher Bereich interpretieren, d.h. die Rotationsgeschwindigkeit fällt proportional zu 1/R^0.5. Das würde aber auch bedeuten, dass es in diesem Bereich keine zusätzliche Masse dazu kommt. Darüber hinaus darf in unserer Umgebung gar keine Dunkle Materie geben.

  115. @Niels danke für die Antwort.
    Es ist mir schon klar, dass „modifizierte Gravitationstheorien“ mehr als nur einzelne Beobachtungen erklären sollten. Aber selbst die (trotz all ihrer Mängel) „Palatini“-Modifikation würde doch nicht konträr zu generellen kosmologischen Theorien/Ansätzen/Beobachtungen sein und der Gravitationslinseneffekt ist doch ein ‚Raumkrümmungs‘-effekt (- oder?). So wie ich das Sotiriou/Faraoni paper verstehe ist doch (zu einem wesentlichen Teil) ’nur‘ die ‚Einfachheit‘ der Formulierung, bspw. Taylorreihenentwicklung, ausschlaggebend für das „Passen“ des Erklärungs-/Beschreibungsmodells. ( … interessant die Links von SCHWAR_A)
    Occam’s razor …. Ja, pragmatisch ist das Erklärungsmodell mit den wenigsten Annahmen zu bevorzugen… aber wenn ich doch die Annahmen der ART sowieso schon habe, benötige ich doch gar keinen weiteren Annahmen um Skalarfeldtensoren in Taylorreihen zu entwickeln, andererseits (wenn ich nicht falsch liege) ist die Annahme von WIMPs / E_8 als Teilchenmodell doch da auch nicht (insbesondere wenn ich den ganzen Theorie Apparat von ART sowieso auch als Voraussetzung habe) von geringerem Komplexitätsgrad. Oder gibt es da tatsächlich komplexitätstheoretische Untersuchungen zu, welches dieser Modelle da eher unter die Occam’s razor Methodologie fällt?
    OLG kam mir nur als zusätzliche Frage, erstmal unabhängig von DM/TeVeS-Derivaten, da hier im thread ja auch Quantengravitationsmodelle angesprochen wurden. Ja ich meinte da das Fermi Gamma-ray Space Telescope. D.h. also bzgl. QLG (od. QSG) ist momentan weder ein eindeutig verwerfender noch belegender Befund bekannt – nochmals Danke für die Antwort.

  116. @SCHWAR_A

    Ok, ist eine EInzelbeobachtung, die zunächst mal unabhängig verifiziert werden muss. Ich erinnere an die überlichtschnellen Neutrinos.

    Wir bekommen ein großes Problem mit der Energiebilanz des gesamten Universums, wenn es tatsächlich keine DM geben würde. Dann dürfte das Weltall nicht flach sein, was es laut Messungen an der Hintergrundstrahlung jedoch ist. Die Lücke ist schwer zu füllen.

    Aber widersprechende Beobachtungen sind natürlich immer bedeutsam und führen oft auf neue Erkenntnisse. Z.B. hatten wir schon mehrere „Alterskrisen“ des Universums, wo Sterne oder die Erde älter waren als das damals aus dem Hubble-Parameter berechnete Alter des Universums. Die dunkle Energie und eine Neukalibrierung der Cepheiden-Veränderlichen (und damit der Entfernung und Leuchtkraft der problematischen alten Sterne) konnten das Problem lösen. Wir kommen nun auf verschiedene Weisen zum gleichen Weltalter, das mit dem der ältesten Sterne kompatibel ist.

    Warten wir mal ab.

  117. @Alderamin et al.
    Die Diskussion darüber sollten wir wohl besser auf den aktuellen Artikel von Florian verlegen… Macht dafür jemand eine Zusammenfassung dort?

  118. @SCHWAR_A

    Den Artikel hatte ich noch nicht bemerkt, als ich obiges geschrieben hatte.

    Der dort zuletzt von Florian verlinkte Artikel von Ethan Siegel fasst den offiziellen Stand sehr gut zusammen. Aber hier können wir das Thema abschließen.

  119. @StefanL

    Gravitationslinseneffekt ist doch ein ‚Raumkrümmungs‘-effekt

    Mir ging es bei diesem Beispiel darum, dass man bei manchen Gravitationslinsen völlig von der ART abweichende Beobachtungen macht. Wenn man dunkle Materie in Betracht zieht passt dagegen wieder alles.

    Es ist mir schon klar, dass „modifizierte Gravitationstheorien“ mehr als nur einzelne Beobachtungen erklären sollten. Aber selbst die (trotz all ihrer Mängel) „Palatini“-Modifikation würde doch nicht konträr zu generellen kosmologischen Theorien/Ansätzen/Beobachtungen sein

    Es gibt momentan aber keinerlei „modifizierte Gravitationstheorie“, die mehr tut als einzelne Beobachtungen zu erklären.

    Die f(R)-Gravitationen und damit auch die Palatini-Gravitation wurden speziell konstruiert, um die Expansion des Universums passend zu den Beobachtungsdaten zu beschreiben, ohne dass dunkler Energie oder dunkler Materie notwendig sind.
    Das schaffen sie auch relativ leidlich.

    Bei den anderen Problemen, die die ART zusammen mit dunkler Materie löst, helfen f(R)-Gravitationen dagegen überhaupt nicht weiter.

    Es ist auch völlig unklar, wie man diese Theorien umbauen müsste, damit das doch noch klappt. Tatsächlich ist sogar unklar, ob es überhaupt möglich ist, diese Theorien derart zu verändern.

    Darüber hinaus weisen diese Ansätze auch noch zahlreiche Mängel auf.
    Da kenn ich mich aber wirklich nicht besonders gut aus, deswegen begnüge ich mich damit, einfach mal zu zitieren und ein paar Sachen zu verlinken, okay?

    1. Due to the structure of the theory, however, Palatini f(R) theories appear to be in conflict with the Standard Model,[2][4] may violate Solar system experiments,[3] and seem to create unwanted singularities.[5]
    2. Bei „f (R) Theories“: 9 Palatini Formalism findet man einiges und bei „9.4: Shortcomings of Palatini f(R) gravity“ kann man weiterlesen
    3. In f(R) gravity: successes and challenges werden außer der Palatini-Gravitation auch die anderen beiden Varianten der f(R)-Gravitation untersucht.

    Die f(R)-Gravitation hilft also ausschließlich bei der Expansion des Universums und weist außerdem ernste Schwachstellen auf.
    Möglicherweise kann man diese Schwachstellen beseitigen und die f(R)-Gravitation so erweitern, dass sie auch die anderen Phänomene richtig beschreibt.
    Ob das klappt, kann aber niemand wissen.
    Deswegen muss man das Ganze momentan auch nicht als Alternative berücksichtigen.

    So wie ich das Sotiriou/Faraoni paper verstehe ist doch (zu einem wesentlichen Teil) ’nur‘ die ‚Einfachheit‘ der Formulierung, bspw. Taylorreihenentwicklung, ausschlaggebend für das „Passen“ des Erklärungs-/Beschreibungsmodells.

    Ich versteh nicht , was du meinst.

    Bei der f(R)-Gravitation geht man vom Ansatz aus, dass die Lagrange-Dichte eine beliebige Funktion des Ricci-Skalars (Krümmungsskalars) ist.

    Bei der Palatini-Gravitation nimmt man zusätzlich an, dass Metrik und linearer Zusammenhang (anders als in der ART) unabhängig voneinander sind.

    andererseits (wenn ich nicht falsch liege) ist die Annahme von WIMPs / E_8 als Teilchenmodell doch da auch nicht […] von geringerem Komplexitätsgrad.

    Worauf willst du eigentlich mit der E8 Lie-Gruppe hinaus? Auf die String-Theorie? Auf die „E8 Theory“ von Lisi? Die hat zumindest irgendetwas mit diesen „248 Teilchen“ zu tun.

    So etwas braucht man aber doch überhaupt nicht.

    Das leichteste supersymmetrische Teilchen wäre zum Beispiel schon ein Kandidat für ein WIMP.
    Ich kenne mich da auch nicht besonders gut aus, aber es gibt ganz bestimmt noch zahlreiche andere theoretische Modelle mit einem WIMP, ohne das man gleich die E8-Keule rausholen muss.

    D.h. also bzgl. QLG (od. QSG) ist momentan weder ein eindeutig verwerfender noch belegender Befund bekannt – nochmals Danke für die Antwort.

    Na ja, wie gesagt, ein verwerfender Fund war gar nicht möglich, weil die QLG noch nicht so weit ist, dass sie einen Zahlenwert voraussagen könnte.
    Das Fehlen eines belegendes Fundes macht die Theorie jedenfalls nicht interessanter. 😉

  120. @Niels
    Palatini f(R) habe ich da nur exemplarisch erwähnt als eine der schwächeren( im Sinne von „mit vielen Mängeln“) Modellansätze die trotzdem schon recht passable ( = leidlich gut ;-))die „kosmologischen Probleme/Problemstellungen“ abdeckt.
    Konflikte mit Bereichen der Teilchenphysik – ja, aber auch mit ’nur‘ ART haben wir kein (verifiziertes oder der Falsifikation hinlänglich überantwortetes) Quantengravitationsmodell, und ART (zumindest in den überprüft und als gut befundenen Teilen) sollte als ‚Grenz-/Spezialfall‘ in modifizierten Gravitationstheorien(mGT) enthalten sein. Und ggfs. sind mGT + Teichenmodell dann noch mächtiger.
    Einzelfallbeschreibungen – nun alpha=alpha(M) in Gleichung (220) in https://arxiv.org/abs/0805.1726 (Sotiriou/Faraoni 2010) bezieht sich insbesondere auf f(R)=R^n Modellansätze. Und es ist auch kaum zu übersehen, daß Faraoni selbst gerne darauf verweist, daß dies alles eher einer Spielwiese entspricht als einer (geschlossenen) Theorie.
    „Deswegen muss man das Ganze momentan auch nicht als Alternative berücksichtigen. nun ja – neben Abell 520 gibt’s ja nun auch –> https://arxiv.org/abs/1204.3924
    Taylor-Reihenentwicklung, u.U. überinterpretiere ich da Gleichung (139) ff in Sotiriou/Faraoni (s.o.) und besser ist vielleicht eine allgemeinere Formulierung wie „Entwicklung höherer Ordnung“.
    WIMPs und E_8 : ich hatte da bislang immer irgendwie eine Kopellung von WIMP und Supersymmetrie im Kopf – aber wenn da (konsistente ;-)) Erklärungsmodelle für WIMPs existieren( die nach wie vor hypothetisch sind) ohne Supersymmetrien werden so Ansätze wie E_8 (Lisi) zumindest in diesem Zusammenhang unnötig.
    Das ändert aber nichts an der Frage, inwiefern Komplexitätsbewertungen dazu führen warum das eine oder andere Erklärungsmodells nach Occam bevorzugt werden sollte.
    A propo Supersymmetrie: (sofern ich nicht falsch liege) impliziert Stringtheorie doch Supersymmetrie aber Supersymmetrie nicht notwendigerweise Stringtheorie?

  121. @StefanL
    Ich jetzt nicht böse gemeint, aber ich finde, deine Text-Formatierung macht es (zumindest mir) ziemlich schwer, deinen Gedankengängen zu folgen.
    Ich weiß deswegen nicht, ob ich richtig verstanden habe, worauf du hinaus willst.

    Wenn sogar die in diesem Bereich Forschenden das Ganze eher als Spielwiese betrachten liege ich doch gar nicht so daneben, wenn ich das als nicht besondern hilfreiche Theorie einschätze, oder?

    nun ja – neben Abell 520 gibt’s ja nun auch –> https://arxiv.org/abs/1204.3924

    Okay, aber ART + DM ist trotzdem die beste Theorie, die momentan zur Verfügung steht.

    Das ändert aber nichts an der Frage, inwiefern Komplexitätsbewertungen dazu führen warum das eine oder andere Erklärungsmodells nach Occam bevorzugt werden sollte.

    Na ja, wie gesagt, wir haben gar keine anderen Erklärungsmodelle die auch nur annähernd Ähnliches leisten.
    Deswegen gibt es da im Moment nichts gegeneinander abzuwägen.

    Und ggfs. sind mGT + Teichenmodell dann noch mächtiger.

    Wer weiß? Das bleibt abzuwarten.

    Ich habe doch nie gefordert, die Arbeit an Alternativen zur ART einzustellen?

    (sofern ich nicht falsch liege) impliziert Stringtheorie doch Supersymmetrie aber Supersymmetrie nicht notwendigerweise Stringtheorie?

    Der zweiteTeil ist richtig.

    Der erste Teil ist prinzipiell falsch, aber wahrscheinlich in der Praxis richtig. 😉

    Stringtheorien müssen nicht supersymmetrisch sein. Deswegen ist es prinzipiell falsch.

    Allerdings beschäftigt man schon seit Jahrzehnten mit Superstringtheorien.
    (Das ist die Kurzform für supersymmetrische Stringtheorie)
    Bei der M-Theorie geht es z.B. ausschließlich um solche Superstringtheorien.
    Deswegen ist es wahrscheinlich in der Praxis richtig.
    Ob es momentan überhaupt noch nennenswert an einfachen Stringtheorien gearbeitet wird, weiß ich nicht. Da kenn ich einfach zu wenig aus. Deswegen wahrscheinlich.
    Soweit ich das überblicken kann beschäftigt sich jedenfalls die deutlich überwiegende Mehrheit der Forscher auf diesem Gebiet nur noch mit Superstringtheorien bzw. der M-Theorie.

  122. Dunkle Materie ist ein abstraktes Konzept das verdinglicht wurde – unzulässig in der klassischen Physik. Es in der realen Welt zu suchen ist so, als wenn man „Sozialismus“ als Konzept physich in der Realität suchen würde – absurd. Es ist eine erdachte Krücke um Einsteins Gravitationsgleichungen zu retten. Dabei kann die Bewegung der Galaxien mit der viel stärkeren Elektromagnetismus erklärt werden – ohne erdachte Rechnegrößen wie dunkle Energie, dunkle Materie etc.

    1. @Walter Orlov: Ja? Der direkte Nachweis von dunkler Materie ist bis jetzt nicht geglückt. Das ist keine große Neuigkeit, das war auch bei vorherhigen Experimenten so. WIe lange so ein direkter Nachweis per Experiment dauern kann zeigt ja auch das Beispiel der Gravitationswellen. Das ändert allerdings nichts an den Fakten die uns sagen, dass sich die Himmelskörper unter dem Einfluss zusätzlicher Materie bewegen.

  123. @Walter Orlov:

    Aber ganz im Gegenteil – Florian Freistetter interessiert sich extrem für Fakten. Was für Fakten haben Sie denn da? Wer oder was ist LUX? Ist das Ganze seriös? Und was für einen Schluss ziihen sich aus dem Ergebnis, dass keine Dunkle Materie gefunden werden konnte?

  124. Tut mir leid, ich bezog mich auf Fortsetzung der Diskussion. Stimmt, es sind schon 4 Jahre her und der Faden verloren gegangen. Zur Erinnerung kann ich zur meinen Webseite schicken:
    https://walter-orlov.wg.am/rotationskurven_und_dunkle_materie/
    Kurz: Ich habe gemeint, dass es Dunkle Materie in Form irgendwelcher Teilchen nicht gibt. Zwar besteht sie aus gewöhnlichen Gestein und Eis. Schließlich wurde mir vorgeworfen, dass ich die Fakten ignoriere. Nun jetzt sehe ich mich bestätigt 😉

  125. Nein, das ist leider zu kurz gedacht. Wenn man etwas sucht und nicht findet, kann das daran liegen, dass das Gesuchte nicht existiert. Oder es war einfach nicht dort, wo man gesucht hat. Oder man hat nicht genau genug gesucht, und bei Materie mit den postulierten Eigenschaften der Dunklen Materie ist das immer die Gefahr Nummer 1.

    Also, noch einmal: Was ist LUX, arbeitet man dort seriös und welche Schlüssen hat man aus dem erzielten Ergebnis gezogen – falls man denn überhaupt schon Schlüsse gezogen hat?

  126. @roel:

    Lass mal stecken – ich habe heute ohnehin keine große Lust, Links zu folgen.

    Halten wir aber fest: Eine seriöse Gruppe von Wissenschaftler hat gerade einen Versuch durchgeführt, Dunkle Materie zu detektieren, war aber dabei nicht erfolgreich. Tja, was für eine Sensation! 😉

  127. @Captain E. Sensationell ist es sicher nicht, aber doch sehr informativ. „gerade einen Versuch durchgeführt“ das ist stark untertrieben.

    „ich habe heute ohnehin keine große Lust, Links zu folgen“. Da kannst du dich selber von der Seriösität überzeugen. Und es ist absolut interessant auch, wenn die Sensation (Nachweis Dunkler Materie) bisher nicht geglückt ist.

  128. Herr Orlov hat es sowieso nicht so mit Belegen, weswegen er vermutlich auch dann die Existenz der dunklen Materie bestreiten würde, wenn das LUX-Experiment welche gefunden hätte. Er behauptet ja auch, dass die Relativitätstheorie Blödsinn sei, obwohl dafür Tonnen um Tonnen an Belegen existieren. Insofern ist eine Diskussion mit dem Herrn eine sinnlose Übung.

  129. @Spritkopf „Er behauptet ja auch, dass die Relativitätstheorie Blödsinn sei“. Ich habe das auf seiner eigenen Homepage überprüft. Danke für den Hinweis.

    Aber das ändert nichts an der Seriösität des „Large Underground Xenon dark matter experiment“.

  130. @roel:

    Mal etwas grundsätzliches: Ein Link in einem Forum oder einem Kommentarboard soll eine Untermauerung der getätigten Aussage sein, dies aber als Angebot, das der Leser annehmen kann oder eben nicht. Ein Link soll aber keinesfalls eine eigene Aussage ersetzen.

    Und ansonsten weiß ich nicht, was genau du für einen Honig daraus ziehen willst. Da hat also eine Forschergruppe keine Dunkle Materie gefunden. Das mag interessant in Hinsicht auf die Art der durchgeführten Suche, also die Methodik und die Genauigkeit. Andererseits hat bislang noch niemand Dunkle Materie nachweisen können. Insofern ist es also nichts besonderes, und diese Forschergruppe wird vermutlich weitermachen, so wie viele andere auch. Wie Florian schon sagte: Gravitationswellen hat man über etliche Jahrzehnte hinweg gesucht und niemals gefunden. Trotzdem hat man immer weiter gemacht. Solange ein negativer Versuch, Dunkle Materie zu finden, nicht mit der Erkenntnis daher kommt, dass man alle Möglichkeiten hinreichend genau abgesucht hat und sie daher fast sicher nicht existieren kann, taugt er eben auch nicht zum Nachweis der Nicht-Existenz.

  131. Spritkopf: „Herr Orlov hat es sowieso nicht so mit Belegen…“

    Wenn Sie etwa ein Haufen von Belegen brauchen, dann passe ich wohl. Aber Herr Einstein selbst meinte, dass nicht alle Belege gleichwertig sind. Jedenfalls werden ihm folgende Worte zugeschrieben:

    „Keine noch so große Zahl von Experimenten kann beweisen, dass ich recht habe; ein einziges Experiment kann beweisen, das ich unrecht habe.“

  132. Captain E.: „und diese Forschergruppe wird vermutlich weitermachen, so wie viele andere auch.“

    Vorerst haben sie aufgegeben (und das ist richtig). Wahrscheinlich ist es nicht billig, die Detektoren stets im Betrieb zu halten.

    Captain E.: „Solange ein negativer Versuch, Dunkle Materie zu finden, nicht mit der Erkenntnis daher kommt, dass man alle Möglichkeiten hinreichend genau abgesucht hat und sie daher fast sicher nicht existieren kann, taugt er eben auch nicht zum Nachweis der Nicht-Existenz.“

    Hier gibt es wohl eine Verwechselung. Ich bin sogar sehr überzeugt, dass Dunkle Materie existiert… aber nicht als Teilchen. Diese Fährte ist falsch und muss lieber früher als später aufgegeben werden.

  133. Dass ein bestimmtes Experiment kein Resultat geliefert hat, belegt nur, dass eine bestimmte Art von Teilchen in einem bestimmten Energiebereich als Teilchen der DM auszuschließen ist. Eben demjenigen Bereich, in dem das Instrument empfindlich war.

    Ganz genau so hat man sich damals dem Higgs-Teilchen angehnähert. Da gab es auch eine Menge Experimente, die als Ergebnis lediglich einen Nichtnachweis erbrachten. Aber ein abwesender Nachweis ist eben kein Nachweis der Abwesenheit.

  134. Ich bin sogar sehr überzeugt, dass Dunkle Materie existiert… aber nicht als Teilchen.

    Können sie diese Hypothese irgend wie belegen?
    Es gibt ja Untersuchungen die diese Idee, jedenfalls größtenteils, eben nicht nahelegen. Kennen sie den aktuellen Forschungsstand?

  135. @Eisentor
    Da müssen Sie mir auf die Sprünge helfen. Etwa MOND-Theorie kann ich als seriös nicht annehmen. Oder meinen Sie das Fehlen Dunkler Materie in Sonnenumgebung?

  136. Eigentlich habe ich meine Hypothese bereits verlinkt…

    Kurz: Rotationskurven von Spiralgalaxien nicht nur flach, sondern weisen merkliche periodische Schwankungen auf, die mit dem Spiralmuster der betroffenen Galaxien übereinstimmen. Also, man müsse davon ausgehen, dass in Spiralarmen Dunkle Materie stärker verdichtet ist als in Zwischenräumen. Deshalb darf Dunkle Materie ähnlich wie sichtbare Materie verteilt sein, d.h. nicht wie HALO, sondern wie galaktische Scheibe. (Die Rechnung zeigt: Auch wenn Geschwindigkeitsschwankungen relativ klein sind, betragen dahinter steckenden Massedichtenschwankungen schon eine Größenordnung.)
    Eine sphärische HALO-Verteilung Dunkler Materie ist für die Annahme, dass diese aus Teilchen, die selbstverständlich durchsichtig sind, ganz gut geeignet.
    Bei einer Scheibenverteilung Dunkler Materie kann man ja auch die Teilchen-Version in Betracht ziehen, aber genauso gut können es Unmengen von Gestein- und Eisbrocken sein. Und in diese Richtung muss man wirklich forschen.

    1. @Orlov: „ber genauso gut können es Unmengen von Gestein- und Eisbrocken sein. Und in diese Richtung muss man wirklich forschen.“

      Wenn da Unmengen Brocken von was auch immer rumfliegen, gibts da auch Unmengen an Staub. Genau so wie die Brocken selbst heizt sich der Staub auf und leuchtet im Infrarotlicht. Die Astronomen kartieren damit seit Jahrzehnten den ganzen Staub der zwischen den Sternen in den Galaxien rumfliegt und wenn da solch enorme Mengen existieren würden um die dunkle Materie zu erklären hätte man das längst gemerkt.

  137. „Unmengen von Gestein- und Eisbrocken“ sind aber, poetisch gesprochen, leuchtende Materie. Nüchtern betrachtet, sind sie damit gerade eben nicht unsichtbar, sondern reflektieren oder absorbieren beobachtbare Frequenzen des elektromagnetischen Spektrums. Gesucht ist aber Materie mit einem Vielfachen der Masse jener Materie, die bereits beobachtet und abgeschätzt werden konnte. Das wird also nicht reichen.

  138. Die Materie muss außerdem nicht-baryonisch sein, denn ansonsten hätte sie die Menge an gebildeten Elementen schwerer als Wasserstoff während des Urknalls beeinflusst. Die relativen Mengen dieser Elemente weisen auf eine bestimmte Dichte von baryonischer Materie während der ersten paar Minuten nach dem Urknall hin. Die Flachheit des Universums wiederum auf eine bestimmte Menge Materie überhaupt. 4/5 dieser Materie können nicht baryonisch sein, sonst würden die obigen Mengenverhältnisse nicht hinkommen. Kometen, Asteroiden etc. in dieser Menge scheiden also aus (und würden ansonsten auch zu weitaus höheren Sternentstehungraten führen).

  139. Ja, klar. Da sind noch die Galaxiehaufen und heiße Gasen, die irgendwie zusammengehalten werden sollen… Alles im Dunkeln. Und genau die oszillierenden Rotationskurven geben uns Hinweis, worum es sich bei Dunkler Materie handelt. Und dieser Hinweis ist so offensichtlich, dass man einfach nicht wegschauen darf.

  140. @Florian Freistetter

    Wir sind eben im Zwischenraum – zimlich genau in der Mitte zwischen zwei Spiralarmen, hier gibt es nicht viel von Dunklem Materie.

    1. @Orlov: „Wir sind eben im Zwischenraum – zimlich genau in der Mitte zwischen zwei Spiralarmen, hier gibt es nicht viel von Dunklem Materie.“

      Das ist Unfug. Wir können den Staub auch noch in fernen Galaxien sehen. Wenn das die dunkle Materie sein soll, hätten wir das gemerkt. Aber sie gehören anscheinend zu den Menschen deren Scheuklappen so fest sitzen, dass sie nichts sehen was nicht mit ihrer jeweiligen Privattheorie der Welt zu tun hat. Sie könnten sich gerne über den aktuellen Stand der astronomischen Forschung informieren. Und darüber nachdenken. Oder halt nicht – aber wenn sie dann lauter Zeug erzählen, dass erwiesenermaßen nicht stimmt, dann dürfen sie sich nicht wundern, wenn man sie nicht ernst nimmt. Oder ignoriert. So wie ich ab jetzt.

  141. @Alderamin

    Nun ja, es gibt die Theorie Primordialer Nukleosynthese. Das weiß ich. Aber aus meiner Sicht ist dies nur die Theorie. Hier mischt sich Kosmologie, als Modell der Entstehung des Universums, in Physik, als Naturwissenschaft, ein. Ich würde doch die Prioritäten setzen 😉

  142. @Walter Orlov

    Nun ja, es gibt die Theorie Primordialer Nukleosynthese. Das weiß ich. Aber aus meiner Sicht ist dies nur die Theorie.

    Eine Theorie ist die höchste Stufe, die ein physikalisches Modell erreichen kann. Die Kugelgestalt der Erde ist beispielsweise eine Theorie. Mehr dazu.

    Damit es eine Theorie ist, muss das Modell überprüfbare Vorhersagen machen und diese Vorhersagen müssen jegliche Versuche der Falsifizierung überstanden haben. Das ist beim Urknall der Fall und betrifft damit auch die primordiale Nukleosynthese.

    Was Ihre Hypothese betrifft, hat Florian bereits erläutert, warum sie leicht falsifizierbar ist. Eine weitere Falsifizierung ergibt sich daraus, dass die Materie der Milchstraße scheibenförmig verteilt ist, die Rotationskurven aber eine kugelförmige Verteilung der Materie nötig machen. Die Rotationskurve einer massiveren Scheibe unterscheidet sich nämlicher außer in der Umlaufzeit überhaupt nicht von der einer weniger massiven. Und wie sollen Asteroiden und Kometen, nicht jedoch Sterne, in den Halo der Milchstraße gelangt sein?

  143. @Alderamin:

    Und nicht vergessen: Asteroiden, Staub und Kometen müssten in einer Anzahl vorliegen, dass ihre gesamte Masse die aller Sterne in der Milchstraße übersteigt. Wenn man sich unser Sonnensystem anschaut, liegt ja bekanntlich 99% aller Materie in der Sonne.

    1. @Captain: „Und nicht vergessen: Asteroiden, Staub und Kometen müssten in einer Anzahl vorliegen, dass ihre gesamte Masse die aller Sterne in der Milchstraße übersteigt. „

      Und der ganze Krempel muss auch irgendwann mal entstanden sein. Wäre wirklich so viel normale Materie beim Urknall entstanden, dann würde die kosmische Hintergrundstrahlung völlig anders aussehen, als sie es tut. Die sieht aber blöderweise genau so aus, als wäre da die gleiche Menge an nicht-normaler, dunkler Materie die man auch aus der Bewegung von Sternen und Galaxien ableiten kann…

  144. @Captain E.

    Richtig, womit dann auch ausscheidet, dass irgendwelche Gasriesen sie in den Halo befördert haben könnten (was schon wegen der Fluchtgeschwindigkeit der Milchstraße nicht hinkommt). Ein Jupiter kann keine 4 Sonnenmassen an Asteroiden aus dem Sonnensystem heraus befördern, ohne lange vorher selbst in die Sonne zu stürzen. Und Asteroiden, Kometen und Co. entstehen nun mal in Sonnensystemen und nicht einfach im leeren Raum, wo die Materiedichte nicht groß genug ist.

  145. @Captain E.

    Sie unterschätzen wirkliche Räumlichkeiten.

    Schätzen wir die Große des Sonnensystems auf Kuipergürtel, heißt 100AE = 4.848*10^-4 pc. Daher ist gesamtes Volumen 1.14*10^-10 pc^3.

    Einfachheit halber lassen wir die Kleinkörper außer Acht. Die Gesamtmasse der Planeten: Merkur 3.3*10^23 kg + Venus 4.87*10^24 kg + Erde 5.97*10^24 kg + Mars 6.42*10^23 kg + Jupiter 1.9*10^27 kg + Saturn 5.69*10^26 kg + Uranus 8.68*10^25 kg + Neptun 10^26 kg = 2.67*10^27 kg.

    Daher ist die Massendichte: 2.67*10^27 kg / 1.14*10^-10 pc^3 = 2.34*10^37 kg/pc^3

    Der mittlere Anstand zwischen den Sternen in der Sonnenumgebung beträgt 2.7pc. Das ergibt das mittlere Volumen, das jedem Stern quasi gehört, von 19.6 pc^3.

    Gehen wir von gleicher Massendichte wie der Planeten im Sonnensystem aus, gelangen wir zur Gesamtmasse der Gesteinsbrocken von 4.59*10^38 kg.

    Die Sonnenmasse 1.988*10^30 kg ist dann nur ein Witz: 100 Millionen kleiner!

  146. Florian Freistetter: „Wir können den Staub auch noch in fernen Galaxien sehen. Wenn das die dunkle Materie sein soll, hätten wir das gemerkt.!“

    Nein. Durch Spektralanalyse wird bei fernen Galaxien festgestellt, dass sie merkliche Menge von schweren Elementen besitzen. In welcher Form sie aber vorkommen, ob nur Staub, oder doch Gemisch aus Staub, Gestein und Wasser, bleib die Sache der Auslegung.

  147. @Walter Orlov

    Gehen wir von gleicher Massendichte wie der Planeten im Sonnensystem aus, gelangen wir zur Gesamtmasse der Gesteinsbrocken von 4.59*10^38 kg.

    Die Sonnenmasse 1.988*10^30 kg ist dann nur ein Witz: 100 Millionen kleiner!

    Öh, öh, Sie haben Captain E.’s Argument nicht verstanden. Auf jede Sonnenmasse sichtbare Materie in der Milchstraße (und anderswo) kommen 4 Sonnenmassen Dunkle Materie. Also auf unsere Sonne schon alleine 4. Und die Sonne ist der absolute Boss im Sonnensystem mit 1000 Jupitermassen. Und Jupiter ist wiederum massiver als der ganze Rest des Sonnensystems zusammen.

    100 bis 300 Milliarden Sterne, aber 1400 Milliarden Sonnenmassen hat die Milchstraße. Das sind so an die 2,8*10^42 kg

  148. @Walter Orlov

    Nein. Durch Spektralanalyse wird bei fernen Galaxien festgestellt, dass sie merkliche Menge von schweren Elementen besitzen. In welcher Form sie aber vorkommen, ob nur Staub, oder doch Gemisch aus Staub, Gestein und Wasser, bleib die Sache der Auslegung.

    Erklären Sie mal bitte:
    – Emissionspektrum vs. Absorptionspektrum
    – Molekularspektren
    – Infrarot-Exzess

    Wenn Sie einem studierten Astronomen Astronomie beibringen wollen… putzig 😉

  149. @Walter Orlov:

    Mal abgesehen von @Alderamins Argumenten ist die Rechnung in #181 grober Unfug.

    Sie nehmen die Dichte der Massen im Sonnensystem, rechnen sie einfach auf ein Volumen von einigen Kubik-Parsec hoch und bilden daraus eine „Gesamtmasse“ die sich in diesem Volumen befinden soll … was für ein Blödsinn …

  150. Tja, so ist es eben! In unserem Sonnensystem befindet sich praktisch nicht, außer der Sonne selber. Sie, ich, Florian Freistetter, alle Kommentatoren, die Erde, die Gasriesen – nichts! Kann es Sonnensysteme geben, wo es anders aussieht? Vielleicht, aber die meisten dürften von der Masseverteilung so aussehen wie unseres. Woher also soll der ganze Krempel kommen? Und wo soll er stecken? Alle Sonnen und Schwarzen Löcher im Universum zusammen genommen, und man braucht trotzdem noch einmal das vierfache an Masse zusätzlich. Das muss schon ein ganz besonderer Stoff sein, dass man ihn nur durch seine geisterhafte Wirkung bemerkt, die seine Gravitation auf die leuchtende Materie ausübt.

  151. @Alderamin

    Es geht selbstverständlich um Infrarot-Exzess. Und? Sollen etwa im Gemisch aus Staub und Gesteine die Gesteine eine andere Temperatur haben? Vielleicht vergessen Sie Kirchhoffsches Strahlungsgesetz?

  152. @PDP10

    Wenn Sie einen unbekannten dunklen Raum betreten, wie wollen Sie wissen, was da drin steht? Die Planeten werden von der Sonne noch beleuchtet, man sieht sie. Mit den Planetioden in Kuipergürtel ist es schon schwieriger. Und danach ist es schon ganz dunkel. Wie wollen Sie wissen, was dort ist?

    Außerdem reicht schon die Massendichte von Asteroidengürtel/Volumen des Sonnensystem, um etwa 10-fache der Sonnenmasse zu bekommen, also, den fehlenden Überschuss an Dunkler Materie.

  153. Ehrlich gesagt, verstehe ich nicht ganz, warum die Wissenschaftler unbedingt nach Pfeife Katholischer Kirche tanzen wollen? Wir sind doch längst in der Epoche der Aufklärung… Aber nein, die Schöpfungsgeschichte muss wissenschaftlich belegt sein! Wen machen sie Gefallen.

    Da gab es so ein verständliches Modell des ewigen Universums. Vom Gestein könnten wir laut dieser Modell so viel wie wollen haben.

  154. @Walter Orlov Warum gehen sie hier von einer gleichmäßigen räumlichen Verteilung aus? Bei der Milchstraße gehen sie ja auch von einer Scheibe aus. Abgesehen davon das man diese Menge an Material sehen würde wenn es gleichmäßig in jede Richtung verteilt wäre…

  155. @Walter Orlov:

    Wenn Sie einen unbekannten dunklen Raum betreten, wie wollen Sie wissen, was da drin steht? Die Planeten werden von der Sonne noch beleuchtet, man sieht sie. Mit den Planetioden in Kuipergürtel ist es schon schwieriger. Und danach ist es schon ganz dunkel. Wie wollen Sie wissen, was dort ist?

    Die Planetoiden werden aber genau so von der Sonne beleuchtet wie die Planeten. Um in ihrem Beispiel zu bleiben. Sie betreten einen dunklen Raum, schauen sich um, nehmen eine Infrarotkamera in Betrieb, einen Geigerzähler und ein Radio. Was Sie dann nicht „sehen“, ist vermutlich wirklich nicht da.

    Außerdem reicht schon die Massendichte von Asteroidengürtel/Volumen des Sonnensystem, um etwa 10-fache der Sonnenmasse zu bekommen, also, den fehlenden Überschuss an Dunkler Materie.

    Das ist leider falsch. Die Massedichte unseres Asteroidengürtels (also der zwischen den Mars- und Jupiterbahnen) ist dermaßen niedrig, dass zusammengenommen ein Himmelskörper herauskäme, der immer deutlich kleiner ist als der Erdmond. In den meisten Gegenden des Sonnensystem ist die Massedichte aber viel geringer. Merken Sie sich einfach, dass 99% der sichtbaren Materie im Sonnensystem in der Sonne selber steckt. Vom Rest (ca. 1%) steckt ein ähnlicher Prozentwert im Jupiter, und der kümmerliche Rest versteckt sich in den Planeten, Monden, Kometen und Asteroiden. Wie wollen sie also in unserem Sonnensystem mit dem ganzen Kleinzeug auf ein vierfaches der Masse der Sonne kommen? Da ist einfach nichts!

  156. @Eisentor
    Die Dicke der Scheibe von Milchstrasse beträgt ca. 1000pc. Wir sind irgendwo in der Mitte und ich rechne nur mit ein paar pc. Man kann also näherungsweise in so kleinem Region von gleichmäßiger Verteilung ausgehen. Es ist klar, dass senktrecht zur Scheibe etwa ein expotentieller Abfall der Massendichte findet.

    Die Rechnung ist sehr grob und muss eigentlich das Gefühl vermitteln, wie groß der Raum zwischen den Sternen ist. Dort kann man alles verstecken.

  157. Captain E.: „Die Planetoiden…“

    So groß sollen die Steine gar nicht sein… Und dann fehlt die Auflösung, diese überhaupt erkennen zu können.

  158. @Walter Orlov:

    So groß sollen die Steine gar nicht sein… Und dann fehlt die Auflösung, diese überhaupt erkennen zu können.

    Die Auflösung hilft Ihnen in keinster Weise weiter. Natürlich werden Sie auf zig Lichtjahre Entfernung keine Steine und keine Staubkörner entdecken können. Was Sie aber fordern, sind Staub, Meteoroiden und Asteroiden mit einer Gesamtmasse des Vierfachen eines Sonnensystems, einer Galaxis, einer Lokalen Gruppe, eines Galaxienhaufens, eines Galaxiensuperhaufens oder eines Filaments. Das wäre extrem viel von diesem hypothetischen Kleinzeug. In der Masse können die Astronomen das nicht einfach übersehen, zumal es ja auch nicht wirklich unsichtbar ist. Es gibt neben der sichtbaren Astronomie bekanntlich noch die mit Infrarot-, Radio-, Röntgen- oder Gammastrahlung. Das ist aber nichts! Die beobachtbare Masse steckt zu 99% in den strahlenden Objekten wie Sonnen und Schwarzen Löchern. Und es gibt zu wenig davon, um alle Effekte zu erklären.

  159. Wer hat dem Herrn Orlov eigentlich den Floh ins Ohr gesetzt, daß ein Sonnensystem eine auch nur ähnliche Massendichte hätte wie der Leerraum zwischen den Systemen?
    Das wäre ja so „realistisch“ wie die Annahme, daß die durchschnittliche Dichte der Erde ähnlich groß wäre wie die Dichte des Erde-Mond-Systems. Oder so.

    Ich sag doch: wo der Herr Orlov auftaucht, werden Facepalms knapp.

  160. Nehmen Sie einen einfachen Pflasterstein und machen Sie Ihre Röntgen- und Gammastrahlung-Untersuchungen. Werden Sie was entdecken? Vielleicht, wenn Sie Ihre Detektoren direkt am Pflasterstein platzieren, aber schon in einer Meter gar nichts.

    Verstehen Sie, Gestein ist das dunkelste, was man sich in einem Universum vorstellen kann. Er ist wie ein kosmischer Chamäleon, der sich zur Umgebung perfekt anpasst.

    Man spricht stets von Staubwolken, aber genauso gut können dies auch Gesteinswolken sein, deren Masse vielfach größer sein darf. Wie wollen sie erkennen, ob die Staubwolken aus Sandkörnern oder aus Steinen bestehen? Kein Teleskop hilft Ihnen.

  161. Mit einem Infrarot Detektor sollte ein Pflasterstein vor einem dunklen Hintergrund relativ einfach zu finden sein.
    Wenn er vor einer Lichtquelle vorbeifliegt sollte es auch möglich sein.
    Schon mal darüber nachgedacht?

    1. @Eisentor: „Mit einem Infrarot Detektor sollte ein Pflasterstein vor einem dunklen Hintergrund relativ einfach zu finden sein“

      Und je kleiner der Krempel ist, desto leichter ist er zu finden. Wenn man den Pflasterstein ordentlich zertrümmert, vergrößert man nämlich die gesamte Oberfläche des Materials und der Staub kann mehr IR abstrahlen als der einzige Stein. Aber was weiß ich schon von Strahlung und solchen Dingen, ich bin ja nur Astronom 😉

  162. @Walter Orlov:

    Nehmen Sie einen einfachen Pflasterstein und machen Sie Ihre Röntgen- und Gammastrahlung-Untersuchungen. Werden Sie was entdecken? Vielleicht, wenn Sie Ihre Detektoren direkt am Pflasterstein platzieren, aber schon in einer Meter gar nichts.

    Steinchen, Sand, Wassertropfen oder gar Wasser- und Quarzmoleküle sind viel zu klein , als dass Sie sie aus einem Flugzeug sehen könnten. Trotzdem erkennen Sie, ob Ihr Flugzeug gerade eine Wüste oder eine Ozean überfliegt.

    Verstehen Sie, Gestein ist das dunkelste, was man sich in einem Universum vorstellen kann. Er ist wie ein kosmischer Chamäleon, der sich zur Umgebung perfekt anpasst.

    Nur in einem Orlovschen Universum – sonst kann man es sehen, wenn nur genug davon zusammenkommt. Und vergessen Sie nicht, dass Sie eine Menge an Staub und Geröll fordern von der vierfachen Gesamtmasse z.B. einer Galaxie. Das ist richtig, richtig viel.

    Man spricht stets von Staubwolken, aber genauso gut können dies auch Gesteinswolken sein, deren Masse vielfach größer sein darf. Wie wollen sie erkennen, ob die Staubwolken aus Sandkörnern oder aus Steinen bestehen? Kein Teleskop hilft Ihnen.

    Ah ja, und wieso können die Astronomen dann sogar die Größe der Partikel bestimmen, die sie beobachten? Anscheinend hilft den Forschern das eine oder andere Teleskop gerade eben doch weiter, nicht wahr?

  163. @Florian Freistetter

    Sie denken ganz richtig: Gesteinswolke muss viel schwerer sein als die Staubwolke, um gleiche Oberfläche zu schaffen. Darum geht es eigentlich.

    Das Zentrum der Milchstrasse ist von dichten Wolken verdeckt. Bestehen sie nur aus feinem Staub oder noch aus Gesten? Direkt können Sie das nicht überprüfen.

  164. Captain E.: „Anscheinend hilft den Forschern das eine oder andere Teleskop gerade eben doch weiter, nicht wahr?“

    Stimmt genau. Zum Beispiel:

    „Die bislang tiefste und flächendeckendste Aufnahme seines Zentrums gelang ihnen mit dem HAWK-I-Infrarotinstrument am Very Large Telescope (VLT) der ESO in Chile. Das Ergebnis ist nicht nur ein Bild von beeindruckender Schönheit, sondern auch eine echte Überraschung. Denn die Aufnahme enthüllte, dass es im Orionnebel eine unerwartet große Zahl an Objekten mit niedriger Masse gibt. Die Forscher fanden zehnmal so viele lichtschwache Braune Zwerge und isolierte Objekte planetarer Masse als bisher angenommen.“
    https://www.wissenschaft.de/home/-/journal_content/56/12054/12497322/Reisen-Erreger-auf-Mikroplastik-durchs-Meer%3F/

    Deswegen bin ich ganz dafür, dass in diese Richtung weiter geforscht wird.

  165. @Walter Orlov:

    Sie denken ganz richtig: Gesteinswolke muss viel schwerer sein als die Staubwolke, um gleiche Oberfläche zu schaffen. Darum geht es eigentlich.

    Das Zentrum der Milchstrasse ist von dichten Wolken verdeckt. Bestehen sie nur aus feinem Staub oder noch aus Gesten? Direkt können Sie das nicht überprüfen.

    Und Sie denken leider nicht richtig. Vor allem lesen Sie nicht richtig. Haben Sie es tatsächlich nicht gesehen? Florian Freistetter hat noch heute hier geschrieben, das die Astronomen sogar die Korngröße bestimmen können.

  166. @Walter Orlov:

    Stimmt genau. Zum Beispiel:

    Ah, Sie versuchen sich gerade in Verbaljudo? Das muss man aber auch erst einmal können.

    “Die bislang tiefste und flächendeckendste Aufnahme seines Zentrums gelang ihnen mit dem HAWK-I-Infrarotinstrument am Very Large Telescope (VLT) der ESO in Chile. Das Ergebnis ist nicht nur ein Bild von beeindruckender Schönheit, sondern auch eine echte Überraschung. Denn die Aufnahme enthüllte, dass es im Orionnebel eine unerwartet große Zahl an Objekten mit niedriger Masse gibt. Die Forscher fanden zehnmal so viele lichtschwache Braune Zwerge und isolierte Objekte planetarer Masse als bisher angenommen.”
    https://www.wissenschaft.de/home/-/journal_content/56/12054/12497322/Reisen-Erreger-auf-Mikroplastik-durchs-Meer%3F/

    Schön, da gibt es also im Orionnebel zehnmal so viele isolierte Objekte und Braune Zwerge wie erwartet. Da stellt sich also zunächst einmal die Frage, wie viel man denn genau erwartet hatte. Nehmen wir einmal an, es wäre ein Prozent der Gesamtmasse des Nebels gewesen. Damit käme man bei günstigster Rechnung darauf, dass zehn Prozent des Orionnebels aus derartigen Objekten besteht. Ihre Forderung wäre aber, dass 80 % des Orionnebels aus Staub, Geröll und schlecht sichtbaren größeren Objekten bestünde. Da sind Sie also noch sehr weit von ihrer Zielmarke entfernt. Außerdem sagt die Häufigkeit solcher Objekte in einem einzigen Nebel absolut nichts über ihre Häufigkeit im gesamten Universum aus. Nicht vergessen, Walter Orlov, Sie fordern das Vierfache der sichtbaren Materie in Form von Staub und Geröll. Glauben Sie ernsthaft, dass die Astronomen dass bei ihren Himmelsdurchmusterungen bislang immer haben übersehen können?

    Deswegen bin ich ganz dafür, dass in diese Richtung weiter geforscht wird.

    Keine Sorge, das wird geschehen! Ihre Myriaden an Geröll wird man damit aber wohl trotzdem nicht finden.

  167. äh, … Herr Orlov:

    Sie denken ganz richtig: Gesteinswolke muss viel schwerer sein als die Staubwolke, um gleiche Oberfläche zu schaffen. Darum geht es eigentlich.

    Das Problem ist, daß Sie Staub von Gestein nicht nur nicht unterscheiden können, sondern daß dieses Kunststück auch sonst niemandem gelingen wird: Gestein ist nämlich nur zusammengebackener Staub.
    Zitat Wikipedia:

    Die chemische Zusammensetzung wird bestimmt, indem man die Elementhäufigkeiten im interstellaren Gas bestimmt und mit den Elementhäufigkeiten in der Sonne vergleicht. Sind bestimmte Elemente im Gas abgereichert, so sind sie höchstwahrscheinlich im Staub kondensiert. Als die Hauptbestandteile des interstellaren Staubs gelten:

    * Silikate, hier insbesondere Pyroxene (MgxFe1-xSiO3) und Olivine (Mg2xFe2(1-x)SiO4).
    * Kohlenstoff in Form von Graphit, jedoch nicht in der typischen planparallelen Schichtung, sondern als Knäuel. Eventuell kommt Kohlenstoff auch in Form von Fullerenen vor.
    * Eise, vorwiegend Wassereis und CO2-Eis.

    Oh, Olivine:

    Olivine sind die häufigsten Silikate und gesteinsbildende Minerale. Sie bilden den Hauptbestandteil des oberen Erdmantels, wo die Magnesium- und Eisenanteile des Olivin etwa im Verhältnis 9:1 stehen, und entstehen in basischen und ultrabasischen intrusiven magmatischen Gesteinen wie Gabbro und Peridotit, aber auch in extrusiven wie dem Basalt. Dunit ist ein intrusives Gestein, das fast ausschließlich aus Olivin besteht und in dem bis zu 15 cm große Forsteritkristalle gefunden wurden.

    Also: nein. interstellarer Staub ist Gestein.

  168. Bullet: „Das Problem ist, daß Sie Staub von Gestein nicht nur nicht unterscheiden können, sondern daß dieses Kunststück auch sonst niemandem gelingen wird: Gestein ist nämlich nur zusammengebackener Staub.“

    Prima, das wollte ich die ganze Zeit sagen. Sie haben mich besser verstanden, als ich mich selbst.

    Der einzige Unterschied zwischen Staub und Gestein ist nur die Masse. Deshalb ist Gestein das beste Vesteck für Dunkle Materie.

  169. @Walter Orlov nochmal kurz.
    Wie verstecken sich ihre Pflastersteine vor Infrarotteleskopen? Und warum stören sie das Licht ferner Sterne nicht?

    1. @Eisentor: „Wie verstecken sich ihre Pflastersteine vor Infrarotteleskopen?“

      Und wo kommen die ganzen Dinger her? Wie viel baryonische Materie kurz nach dem Urknall vorhanden war wissen wir ja aus der Beobachtung des CMB… und das reicht nicht für die dunkle Materie.

  170. Ich hoffe das Herr Orlov selber die Probleme seiner Hypothese findet. Darum versuche ich es mit den einfachsten Dingen die mir auffallen.

  171. @Walter Orlov:

    Prima, das wollte ich die ganze Zeit sagen. Sie haben mich besser verstanden, als ich mich selbst.

    Ihre Versuche in Verbaljudo sind immer noch beklagenswert schwach.

    Wir können Ihnen im übrigen nur ganz warm ans Herz legen, wirklich zu lesen und zu verstehen, was wir versuchen, Ihnen nahezubringen.

    Der einzige Unterschied zwischen Staub und Gestein ist nur die Masse. Deshalb ist Gestein das beste Vesteck für Dunkle Materie.

    Genau deshalb ist Gestein gerade eben kein guter Kandidat für Dunkle Materie. Der Grund: Es ist sichtbar. Man kann es sehen. Es reflektiert Licht, es absorbiert Licht und es sendet sogar Licht aus. Warum in aller Welt wollen Sie das nicht begreifen?

  172. Eisentor: „Darum versuche ich es mit den einfachsten Dingen die mir auffallen.“

    Dann versuche ich auch: Kennen Sie sich mit den Strahlungsgesetzen oder mit der Physik überhaupt aus?

  173. Genug um zu wissen das Staub sichtbar ist (zumindest im Infrarot) und Licht (jedenfalls bestimmte Wellenlängen, abhängig von „Art“ des Staubs) absorbiert.

  174. @Eisentor

    Schlagen Sie schon nach?

    Dann weitere Fragen:

    Wie heiß sollen die Pflastersteine sein, um im Infrarotbereich zu strahlen? (Stichwort: Wiensches Verschiebungsgesetz)

    Wo soll die Energie herkommen, um die Pflastersteine so weit aufzuhezen? (Haben sie etwa einen versteckten Kernreaktor drin?)

  175. Schlagen Sie schon nach?

    ne. Der Beitrag wird noch moderiert.

    Welche Temperatur haben den ihre Pflastersteine das sie mit Infrarotteleskopen nicht gefunden werden können? Wissen sie was in der Infrarotastronomie möglich ist?

    1. @Alle: Falls euch diese sinnlose Diskussion nicht ganz dringend Spaß macht, würde ich überlegen, sie zu beenden. Erfahrungsgemäß bringt sowas nix, außer großer Genervtheit. Herr Orlov ist (wie viele andere „Forscher“ mit Privattheorie) so sehr in der Vorstellung gefangen, er hätte etwas entdeckt was kein anderer entdeckt hat, dass er nicht bereit ist, über den Tellerrand seiner eigenen Gedanken hinaus zu blicken. Es bringt also auch nix, auf die vielen Fehler und fehlerhaften Vorstellungen seiner „Theorie“ hinzuweisen. Am Ende wird er weiterhin daran glauben (den Fakten, die den Glauben ersetzen könnten gibt es ja nicht). Ich sage das vor allem auch deshalb, weil ich mich ab heute Abend auf einen Kurzurlaub begebe und die Moderation der Kommentare schwierig wird bzw. lange dauert…

  176. Oh, okay, dann mein letzter Kommentar.
    @Orlov:

    Der einzige Unterschied zwischen Staub und Gestein ist nur die Masse.

    Nein. Es gibt keinen Unterschied. 500 000 Tonnen Staub, die im Raum herumfliegen und 500 000 Tonnen Gestein, die feinverteilt im Raum herumfliegen, sind identisch.
    Und zum Thema „Infrarotstrahlender Staub“, wieder aus dem Wiki-Artikel von weiter oben:

    Typische Staubtemperaturen liegen im Bereich von 10 bis 100 Kelvin. Auf galaktischer Skala kann man zwischen zwei „Staubpopulationen“ unterscheiden, nämlich einer kalten und einer warmen Komponente, wobei die Temperaturen bei 10-20 Kelvin bzw. bei über 30-100 Kelvin liegen. Der kalte Staub dominiert dabei die Massenbilanz und ist in Spiralgalaxien meist sehr weit ausgedehnt, ähnlich dem neutralen Wasserstoff. Er macht sich aufgrund der niedrigen Temperatur durch Emission im schwerer zugänglichen, fernen Infrarot ab ca. 100µm bemerkbar. Der warme Staub hingegen ist weniger diffus verteilt und kann mit Sternentstehungsgebieten wie etwa Emissionsnebeln und kalten Molekülwolken assoziiert werden. Denn junge, neu gebildete Sterne emittierten jene starke UV Strahlung, welche für die Erwärmung des interstellaren Staubes verantwortlich ist.

    Jaja. Für 20K heißen Staub muß man schon ordentlich heizen…

  177. @Walter Orlov:

    Spektrum.de hat extra für Sie ein schönes Lestestück über Staub. Was das ist, wie man ihn ’sieht‘ usw.:

    https://www.spektrum.de/magazin/kosmischer-staub/822829

    Ansonsten stimme ich @Florian zu:
    Es macht keinen grossen Spass mit jemand über Astronomie und Physik zu diskutieren, der einerseits in einem solch herablassenden Tonfall schreibt, andererseits aber offenbar keine Ahnung hat, wie man an Ihrer Milchmädchenrechnung oben sieht.
    Denken Sie wirklich, die Materiedichte in der direkten Umgebung eines Sterns lässt sich hochrechnen auf den riesigen Raum zwischen Sternen?
    Und denken Sie ernsthaft, wenn da soviel Materie wäre (Pflastersteine, Staub, Bier, was auch immer), wäre das noch niemand aufgefallen?

    @Florian: Ich weiss, ich weiss … aber eigentlich ist dieser Post hier nur dazu da, den wirklich schönen Artikel von Spektrum.de hier zu verlinken :-).

  178. Lesestück!

    Ja wie, PDP10, weil doch Staub sich zB hier in der Wohnung aus lauter Resten bildet, hielt ich das für den neuen Artikel dieses chinesischen Kosmologen. Kombiniert mit dem klassischen SprachVorurteil.

  179. Ich denke wenn der Teilchenbeschleuniger Inbetriebnahme explodiert also wenn der Beschleuniger eine Beschädigung im Kernsystem hat, die, die energiereiche dunkle Materie freilässt dann fliegt überall Dunkle Materie und man könnte unbekannte Fähigkeiten bekommen wie z.B in Lichtgeschwindigkeit rennen es kommt drauf an welche Fähigkeit man bekommt .

  180. @Unbekannt:
    Mmh ja, echt verwirrend. Beim Beschleuniger scheint eine spezielle Unterart von dunkler Materie zu entstehen. Auf den Aufnahmen der Explosion sieht man ja, dass die auch nicht dunkel ist, sondern so rot-orange leuchtet. Scheint jedenfalls keine kosmische DM zu sein. Es gab da vor längerem auch mal so einen Zeitschriften Artikel, auf den auch der Florian hier hingewiesen hatte. Nach dem sollte die Erde mit einer Wolke aus DM kollidieren. Ich hab in der betreffenden Nacht die ganze Zeit mit ausgebreiteten Armen auf einem Kornfeld gestanden. Es wurde zwar richtig dunkel, aber Superkräfte hab ich keine bekommen…

Schreibe einen Kommentar zu Captain E. Antworten abbrechen

Deine E-Mail-Adresse wird nicht veröffentlicht. Erforderliche Felder sind mit * markiert

Diese Website verwendet Akismet, um Spam zu reduzieren. Erfahre mehr darüber, wie deine Kommentardaten verarbeitet werden.